Pharm Exam 2 BLUEPRINT - STUDY!

¡Supera tus tareas y exámenes ahora con Quizwiz!

A client has been on an antibiotic for two weeks for treatment of an infection. The client asks the nurse why a superinfection has been caused by this medication. What is the nurse's best response?

"Antibiotics can disrupt the normal bacteria in your body, causing another infection to occur."

The health care provider is preparing to order rifampin and pyrazinamide for a female client with active tuberculosis. What question should the provider ask this client before confirming this order?

"Are you pregnant?"

What statement made by the patient indicates to the nurse that the patient does not understand the transmission of HIV?

"As long as I am taking the HAART therapy, I cannot transmit the virus to anyone else."

A client has been prescribed INH for the treatment of tuberculosis. The nurse teaches the client about dietary restrictions while taking this medication. What is the most important instruction?

"Do not drink beer or red wine while taking this medication because a serious adverse reaction can occur."

After teaching a client who is prescribed oral erythromycin, the nurse determines that the teaching was successful when the client states which of the following?

"I should drink a full 8-oz glass of water when I take the medicine."

The nurse is teaching a male client with HIV about his new antiviral drug regimen. Which client statement would suggest that the teaching plan was effective?

"I should expect some nausea and vomiting."

Which statement by a client taking a sulfonamide requires further instruction?

"I will make sure to use extra sunscreen when I go to the tanning booth."

An adult client is being treated for genital herpes with acyclovir. Which client statement indicates a need for further health education?

"It's a relief to know that this drug will stop me from spreading the infection."

The clinic nurse is responsible for teaching a client about a new prescription for doxycycline. The nurse should increase which instruction in the teaching?

"Limit your exposure to the sun."

The nurse is caring for a client whose prescribed course of cefaclor will soon be completed. What health education should the nurse provide to the client?

"Make sure to avoid drinking any alcohol for the next three days."

A 16-year-old has come to the clinic and asks to talk to a nurse. The nurse asks the teen what she needs and the teen responds that she has become sexually active and is concerned about getting HIV. The teen asks the nurse what she can do keep from getting HIV. What would be the nurse's best response?

"Other than abstinence, only the consistent and correct use of condoms is effective in decreasing the risk of getting HIV."

The nurse is reviewing the medication instruction for the client taking acyclovir. Which statement by the client would indicate the need for additional teaching?

"This medication will maintain the symptoms and cure my disease."

A nurse is teaching a client with herpes zoster to apply acyclovir ointment. What guidance should the nurse include in the instructions?

"Wear gloves when applying the ointment."

The client has been taking a fluoroquinolone and now reports that he has a white patch in his mouth. What is the best response of the nurse?

"You may be experiencing an additional infection. I will discuss this with your health care provider."

Cephalosporin drugs include

"cepha" cephalexin (Keflex) (1st generation)

Penicillin and penicillinase-resistant drugs include

"cillins" ampicillin, amoxicillin (Amoxil)

Tetracycline drugs include

"cycline" minocycline (Arestin, Minocin)

Patient teaching for aminoglycosides:

- Take full course of medication - Provide safety measures if CNS effects occur - Small frequent meals, frequent mouth care, ice chips or sugarless candy if stomatitis or mouth sores are an issue - Tell patient to stay hydrated to avoid renal toxicity and maintain nutrition - Advise about dosages and regimen and possible adverse effects - Avoid exposure to other infections (large crowds or sick people) - Report difficulty breathing, severe headache, loss of hearing or ringing of ears, and changes in urine output

What are somethings you should implement in caring for your patient taking systemic antifungal medication?

- comfort and safety provisions if CNS effects occur - antipyretic for fever and chills

Prophylaxis:

-People traveling to an area where malaria is endemic -Patients who are undergoing GI or genitourinary surgery -Patients with known cardiac valve disease, valve replacements, and other conditions requiring invasive procedures (bacteria can build up in new heart valve causing more problems)

0.33% NS

0.33% saline in water (1/3 normal saline)

1 kg

1,000 grams

What are some antineoplastic drugs that are commonly used today?

1. Alkylating agents 2. Antimetabolites 3. Antineoplastic antibodies 4. Mitotic inhibitors 5. Hormones and hormone modulators 6. Cancer cell specific agents 7. Protein tyrosine kinase inhibitors

Antineoplastic drugs can work by affecting what?

1. Cell survival 2. By boosting the immune system in its efforts to combat the abnormal cells

What are the two groups cancers are divided into?

1. Solid tumors 2. Hematological Malignancies

steps of IV push medications

1. calculate dose in mL 2. calculate safe rate of administration

1 Ib = ? oz

16 oz

Needle gauge and length

18-27 gauge 3/8th to 2 inches

intramuscular needle

19-24 gauge 1-2 inch length Typical: 21 gauge, 1 1/2 length needle

15ml=

1tbsp

Amebiasis has how many life cycles?

2 1) cyst, dormant stage. can live outside or in intestines 2) trophozoite stage in ideal environment - large intestine in humans

A client comes to the health care facility reporting flulike symptoms. After a thorough assessment, the client is diagnosed with influenza and is to receive oseltamivir. The nurse understands that this drug has been prescribed because the client been symptomatic for less than:

2 days

A person is said to be cured of cancer when he or she has been cancer free for a period of __________ years.

5 (Rationale: When a person has been cancer free for 5 years, that person is said to be cured of cancer.)

1 tsp = ___ mL

5 mL

How long does it take for a patient to be "cured" from cancer?

5 years due to the possibility that cancer cells will emerge from dormancy

D5W

5% Dextrose in Water

D5W

5% dextrose in water

A patient is receiving an antibiotic IVPB in 50 mL q8H to run over 1 hour plus a maintenance IV of 100 mL/ hour. What is the 24 hour parenterally intake?

50 x 3 = 150 (24 hours divided by 8 = 3, that's 3 times 50 mL was infused in 24 hours) 100 x 21 = 2100 (24-3 times it was given) answer is 2250

1 pt=___mL

500 mL

500mL NS IV; infuse at 75mL/hour

500mL/75mL/hour=6.67

500mL D5 1/2NS with 20mEq KCl IV to run 75mL/hr on a pump

500ml to infuse at 75mL/hr

180 mL

6 oz

Penicillinase-resistant is

A group of drugs that remain effective against bacteria that are now resistant to penicillins

Aminoglycosides

A group of powerful antibiotics used to treat serious infections caused by gram-negative aerobic bacilli

People have different levels of risk for becoming infected with HIV. Which client is more at risk of becoming infected with HIV?

A person having casual intercourse with multiple partners

Fluroquinolones are

A relatively new synthetic class of antibiotics with a broad spectrum of activity

A client receiving isoniazid (INH) and rifampin has a decreased urinary output and decreased sensation in the great toes. Which laboratory values should be assessed?

ALT and AST

Antifungal drugs function to do what?

Alter cell permeability of the fungus, leading to cell death

Metronidazole is used to treat what?

Amebiasis

If your patient seemed to resistant to intestinal invasion but passed cysts in still, what infection may they have? What would you treat them with?

Amebiasis Metronidazole

A client diagnosed with infective endocarditis would be most effectively treated with which medication?

Ampicillin

In general, progression from first-generation cephalosporins to fourth-generations results in which of the following? Select all that apply.

An increase in sensitivity of gram-negative microorganisms. A decrease in the sensitivity of gram-positive microorganisms.

A client who is HIV positive is taking zidovudine. Which adverse effects should the nurse closely monitor for in this client?

Anemia and granulocytopenia

Nursing Considerations for Fluroquinolones:

Assess: - Known allergies, history if myasthenia gravis, history of renal disease, pregnancy or lactation - Skin - Perform culture and sensitivity, BUN and creatinine test Implementation: - Check culture results - Monitor renal function tests - Monitor site of infection and presenting signs and symptoms - Provide small frequent meals and frequent mouth care - Drink lots of fluids - Avoid UV light or sun exposure - Report headache, difficulty breathing, diarrhea, skin rash, fainting spells, heart palpitations, tendon pain, weakness

protease inhibitors

Atazanavir Darunavir Fosamprenavir Indinavir Lopinavir Ritonavir nelfinavir

Which of the following would a nurse identify as being classified as a macrolide?

Azithromycin

A client with a positive sputum culture for TB has been started on streptomycin antitubercular therapy. Upon review of the laboratory results, the nurse notes that the client may be experiencing toxicity if which of the following results is abnormal?

BUN and creatinine

Aminoglycosides are

Bactericidal and treat gram-negative aerobic bacilli

To prevent viral infections, what precaution should the general public take?

Become vaccinated against prevalent virus infections.

Macrolides

Bind the subunit of ribosome within the bacterial cell and interfere with protein synthesis in susceptible bacteria

Fahrenheit to Centigrade

C= (F-32) / 1.8

For BG 251-300

CALL THE DOCTOR!

The nurse should recognize that maraviroc belongs to what category of antiretroviral?

CCR5 antagonists

The nurse is caring for a client receiving antiviral medications to treat HIV. What laboratory study should the nurse monitor most closely to determine the effectiveness of the medications?

CD4 count

Calculating Basic IV Drip Rates

Calculation Number of mL to infuse x TF ----------------------------------- = Drops per min (gtt/min) Number of minutes to infuse

Example: Order: S/S (sliding scale insulin) Blood Glucose result: 340

Calculation: 340-100 / 40 = 6 units administer 6 units of insulin always round insulin to the nearest whole number

A client has a history of a life-threatening anaphylactic reaction to penicillin G. Which medication should not be administered to this client?

Cefadroxil

After teaching a group of students about carbapenems, the instructor determines the need for additional teaching when the students identify what as an example?

Cefuroxime

Beta-lactam antibiotics, such as penicillins and cephalosporins, fight infection by inhibiting development of the causative bacteria. What specific component development do these drugs affect?

Cell wall synthesis

Many antiprotozoal agents have not proven safe in what age group?

Children

The health care provider has prescribed ciprofloxacin. For which client would you contact the provider to clarify the order because the medication is contraindicated? (Select all that apply.)

Children younger than 18 Pregnant and lactating mothers

What is the MAIN antimalarial used today?

Chloroquine

Which drug would a nurse recommend a woman keep on hand who struggles with vaginal infections?

Clotrimazole

A patient diagnosed with acute primary HIV infection is in the clinic. What treatment should be initiated for this patient?

Combination antiretroviral therapy

You assess a patient with moist, small, white colonies that extend down the phaynx. What should you do to determine treatment for this patient?

Culture Sensitivty test Assess for allergy Perform Physical assessment inspect area of application for color, temperature and evidence of lesions to establish baseline

Clients with tuberculosis that do not respond to the primary anti-TB drugs such as INH, may need to undergo therapy using second-line defense drugs. Which medication is a second-line drug?

Cycloserine

A client who takes zinc daily is diagnosed with a severe infection and is ordered levofloxacin (Levaquin). The nurse is aware that taking these two drugs may have what affect on the antibiotic?

Decreased absorption

What are common adverse effects associated with antifungals?

Dermatological reactions, GI effects, Renal damage, Liver damage, bone marrow suppression

Fungai that are used to be treated by topical agents cause a mycoses called what?

Dermatophytes

Dilution Calculations

Diluting tube feeding involves mixing the nutritional supplement with a specific amount of water. the most common dilutions are: 1/2 strength 1 / 4 strength (one quarter) 1/3 strength (three quarter)

A client has been prescribed ciprofloxacin IV for the treatment of cellulitis. After initiating the infusion of the client's first scheduled dose, the client develops a pronounced rash on the chest and arms. How should the nurse respond initially to this event?

Discontinue the infusion and inform the health care provider promptly.

A client has been treated with abacavir for the past 6 weeks. The client contacts the physician's office with reports of diarrhea, abdominal pain, sore throat, cough, and shortness of breath. Which is the appropriate action to take for this situation?

Discontinue the therapy.

Drug interactions

Diuretics, neuromuscular blockers, succinylcholine, or citrate anti coagulated blood

Antineoplastic agents

Drug used to combat cancer or growth of neoplasms

A client is receiving nevirapine as part of a treatment for HIV infection. The nurse would instruct the client about which adverse effects as most commonly experienced?

Dry mouth and dyspepsia

There are many antiviral medications on the market. What is the clinical reason for this?

Each medication is only effective against a small number of specific viral infections.

A client being treated for cellulitis with a cephalosporin asks what the essential difference is between generations of this medication. The nurse should respond to the client's question based on what fact?

Each successive generation is more effective against gram-negative microorganisms.

Narrow Spectrum of Activity:

Effective against only a few microorganisms with a very specific metabolic pathway or enzyme

fusion inhibitors

Enfuvirtide

The nurse is caring for a client diagnosed with Legionnaires disease. What medication would the nurse expect the health care provider to prescribe?

Erythromycin

Labeling Ivs

Every Iv must be labeled so that any professional can check both the fluid that is being infused and the drip rate. A typical order includes the following information: *patient name *room *bed number *date *time

Centigrade to Fahrenheit

F= (C * 1.8) + 32

Locally active antiviral agents can be applied to open lesions.

False

True or False: A male who has trichomoniasis typically has signs and symptoms of it when transmitting it.

False (Rationale: A male with trichomoniasis typically has no signs or symptoms. Women will present with reddened, inflamed vaginal mucosa, itching, burning, and a yellowish-green discharge.)

True or False: Amphotericin B is contraindicated for use during pregnancy.

False (Rationale: Amphotericin B has been used successfully during pregnancy, but it should be used cautiously and only if benefits outweigh risks.)

True or False: The incidence of fungal infections has recently dropped due to advances in drug therapy.

False (Rationale: The incidence of fungal infections has increased with the rising number of immunocompromised individuals.)

True or False: The organism that causes amebiasis exists in five stages.

False (Rationale: The organism that causes amebiasis exists in two stages: a cystic dormant stage and a trophozoite stage.)

Ketoconazole is excreted where?

Feces (hepatic impairment is adverse effect)

What is the brand name for Metronidazole

Flagyl

protease inhibitor adverse effects

GI effects, changes in liver function, increased cholesterol and triglyceride level, steven johnson syndrome risk

nonNRTI adverse reactions

GI related, dizzy, blurred vision, headache

The nurse is caring for a client with AIDS who has developed cytomegalovirus (CMV). The nurse anticipates the health care provider will order which drug to treat this client?

Ganciclovir

What are some causes of cancer?

Generics (breast cancer for example), viral infection, constant irritation, cell turnover, and stress

Which of the following drugs would be classified as an aminoglycoside?

Gentamicin

The tetracyclines are effective against a wide range of which type of organisms?

Gram-positive and gram-negative organisms

Your client is currently taking aminoglycosides. You know that this type of medication is used in all of the following microorganisms, EXCEPT:

Gram-positive cocci.

Alopecia

Hair loss; common with antineoplastic drugs

What should you monitor in a patient taking Anidulafungin?

Hepatic The echinocandins are known to be extremely toxic to the liver

Geriatric patients are at increased risk for what?

Hepatic toxicity Dosage may need to lowered

A client is diagnosed with latent tuberculosis infection. The nurse expects that the treatment plan will include which drug?

INH

What to do if phlebitis occurs

IV must be discontinued and another site found

A 58-year-old with HIV is starting treatment with Combivir. He currently has a CD4+ cell count of less than 200 cells/mL and a viral load greater than 45,000 copies/mL. The nurse treating the client knows that what is a sign of effective drug therapy?

Increased CD4+ cell counts

A nurse is caring for a patient who is being administered penicillin. What are the common adverse reactions to penicillin a nurse should assess for?

Inflammation of the tongue and mouth

Macrodrip

Infusion sets without a small needle the package label will always state the drops per mL for this infusion set.

Actions

Inhibits protein synthesis in susceptible strains of gram-negative bacteria causing cell death

A school nurse is talking with a health class of freshman high school students about human immunodeficiency virus (HIV). What patient would the nurse tell the students is at the greatest risk of contracting HIV?

Injecting drug user

A patient who has been on penicillin therapy for several days has developed inflamed oral mucous membranes and swelling in the tongue and the gums. The primary health care provider has diagnosed it as a fungal superinfection of the oral cavity resulting in impaired oral mucous membranes. Which of the following interventions should the nurse perform?

Inspect mouth and gums regularly.

A client's upper respiratory infection is believed to be bacterial so the health care provider has prescribed amoxicillin 500 mg PO q8h. How should the nurse best interpret this prescription?

It aligns with the recommended dose, route and indication for this medication

Why should Chloroquine be avoided during pregnancy?

It can cause birth defects

A client is prescribed penicillin V orally for a strep throat. What is the mechanism of action of this medication?

It inhibits cell wall synthesis.

In providing care to a client with HIV, the nurse knows that reverse transcriptase, an enzyme found in retroviruses, affects RNA and DNA in which way?

It turns RNA into DNA, which helps reproduce more of the virus.

Which antifungals are known to imapir hepatic function in older patients?

Ketoconazole Itraconazole Griseofulvin

Determining Hours an IV will Run

Knowing how long an IV will last will prepare you for whether the IV is moving to fast or slow, or whether you need to prepare the next IV Number of mL ordered -------------------------- Number of mL per/Hr.

What are some examples of a hematological malagnancy?

Leukemia's and lymphomas which occur in the blood forming organs

The nurse is caring for a 6-year-old girl who has been diagnosed with active tuberculosis. The physician is unlikely to prescribe which of the following drugs because its use is not recommended in children?

Levofloxacin

A client is receiving rifampin and isoniazid in combination for treatment of tuberculosis. What would the nurse need to monitor closely?

Liver function studies

Autonomy

Loss of normal controls and reactions that inhibit growth or spreading

What do many patients experience while taking these drugs and why?

Many experience nausea and vomiting due to the toxic drugs they are taking and the body's response to the elements of cell death circulating in the bloodstream

What are common adverse effects of Metronidazole?

Metallic/unpleasant taste headache, dizziness, nausea, ataxia

Numerous residents of a long-term care facility have developed Clostridium difficile-associated diarrhea over the past week. The nurses at this facility would anticipate that many residents would require which medication therapy?

Metronidazole

How is the risk of renal impairment best minimized when foscarnet is administered?

Monitoring renal function two or three times weekly during induction

A patient is receiving treatment for HIV that includes zidovudine therapy. The patient is experiencing proximal muscle weakness as well as elevated creatinine kinase values. To what adverse effect does the nurse recognize these findings as attributable

Myalgia

Leading cause of death from infectious disease is

Mycobacterium tuberculosis

Any infection with a fungus is called a ___________.

Mycosis

The nurse is caring for a client who is on sulfonamide therapy and monitors the client for which common side effects? (Select all that apply.)

Nausea Diarrhea Photosensitivity

What are common side effects related to the release of merozoites

Nausea, malaise, shaking, fever, chills

A client with a diagnosis of HIV has impaired renal function due to a concurrent diagnosis of diabetic nephropathy. What medication is most appropriate to treat this client's HIV?

Nelfinavir

Neoplasm

New/cancerous growth; when abnormal cells can multiply and grow

Order: 1000 mL 1/2 NS iv 8am-8pm

No math is needed. the Iv will last for 12 hours

NS

Normal (or isotonic) saline

Calculating Drip Rate

Number of milliliters to infuse X DF/ Number of minutes to infuse= Drops per minute or gtt/minute

The nurse is assisting in the admission of a client with a suspected urinary tract infection and an oral temperature of 100.9°F. The health care provider has written the following orders: acetaminophen 500 mg PO for elevated temperature; urinalysis for culture and sensitivity; sulfasalazine (Azulfidine) 500 mg PO four times daily; and force fluids to 3 L/day. Which order will the nurse complete first?

Obtain urine for culture and sensitivity.

What type of bandages must be avoided when using Clotrimazole?

Occlusive bandages

A nurse is preparing to administer famciclovir to a client with a herpes virus infection. The nurse should expect to administer this drug by which route?

Oral

Local infections include what types?

Oral or vaginal yeast infection Tinea infections (athletes foot, jock itch)

A group of students are reviewing information about antiviral agents used to treat influenza and respiratory virus infections. The students demonstrate understanding of the material when they identify which drug as appropriate for treating Avian flu?

Oseltamivir

In which of the following patients is the use of cidofovir contraindicated?

Patients with renal impairment

Cephalosporins are structurally and chemically related to which classes of antibiotics?

Penicillins

When a Supply is a Percent

Percents in the metric system are always interpreted as GRAMS PER mL example: 2% means 2 g per 100 mL

A HIV-positive patient is being treated with didanosine as part of the antiretroviral therapy. Which of the following symptoms should the nurse monitor for and immediately report to the care provider?

Peripheral neuropathy

A client has been prescribed lomefloxacin (Maxaquin) for a respiratory infection. What must the nurse closely monitor for in this client?

Photosensitivity

You should mark a calendar for _________ doses.

Prophylaxis Start 4 weeks in advance

A client who is hospitalized and receiving antiretroviral therapy has a nursing diagnosis of Risk for Injury related to weakness and dizziness. Which would be most appropriate for the nurse to do?

Provide for frequent rest periods.

A client with a diagnosis of necrotizing otitis externa has been prescribed ciprofloxacin. What microorganism is the most likely cause of the disorder?

Pseudomonas aeruginosa

What was the FIRST antimalarial drug?

Quinine

Insulin Syringes

Remember that there are two types of syringes for insulin: 100 unit ( 1 mL syringe ) and low dose insulin syringe that goes to 50 units. depending on the amount you are injecting to the patient depends on whether or not you have to use either / or Refer to pages 168-169 for examples

A client is started on sulfamethoxazole-trimethoprim for a urinary infection. What adverse effect should the nurse assess with this client?

Renal damage

A 70-year-old man is being treated for herpes zoster virus. He has been prescribed acyclovir (Zovirax). The clinic nurse should prioritize assessments of which of the following?

Renal function

The protozoa that cause malaria have deveopled what over the past years?

Resistance to antimalarial drugs

A client is receiving zanamivir. When describing this drug, the nurse understands that it is absorbed through which mechanism?

Respiratory tract

Chloroquine is known to have what major adverse effects?

Retinal damage Ototoxicity Pruiritis Loss of hair

The nurse is caring for a child with respiratory syncytial virus (RSV). Which drug should the nurse expect the pediatrician to order?

Ribavirin

The nurse knows that pregnant caregivers should not inhale which medication while administering the medication to clients?

Ribavirin

A middle-aged patient has been prescribed tetracycline as part of his Prevpac for the treatment of H. pylori. The patient has a history of atrial fibrillation which is being treated with digitalis drugs. Given his history and current medications, the patient is at risk for which of the following conditions?

Risk of toxicity

A patient with TB is admitted to a health care facility. The nurse is required to administer an antitubercular drug through the parenteral route to this patient. Which of the following precautions should the nurse take when administering frequent parenteral injections?

Rotate injection sites for frequent parenteral injections.

Example: Order: 500 mL NS IV ; infuse at 75 mL/hour

Run for 6.7 hours or 6 hours and 42 minutes (take .7 x 60 = 42 minutes) 500 mL ---------- = 6.7 Hours 75 mL

A client is receiving a cephalosporin and an aminoglycoside as combination therapy. What assessment should the nurse prioritize?

Serum BUN and creatinine levels

A 43-year-old man has been diagnosed with active TB. He is prescribed a multiple drug therapy, including INH and rifampin. A priority assessment by the nurse will be to monitor which combination of laboratory test results?

Serum alanine transaminase, aspartate transaminase, and bilirubin

An elderly female client is admitted to the medical floor with pustules on her body that travel along the nerve route in her legs and arms. The health care provider prescribes the drug acyclovir (Zovirax). What disease is this client demonstrating?

Shingles

Sliding Scale Regular Insulin Dosages

Sliding scale insulin orders refers to a method of insulin administration that is based on thebblood glucose result.

What should you advise a patient to do if they notice a rash while taking Clotrimazole?

Stop the medication. This is a sign of hypersensitivy

Why can stress lead to cancer?

Stress suppresses the activities of the immune system so if a cell is mutating while someone is under prolonged stress it is more likely going to grow into a neoplasm because the immune system is not fully active

A young adult client's acne has responded well to treatment with tetracycline. However, the client has now returned to the clinic 6 weeks later with signs and symptoms of oral candidiasis. The nurse should recognize that this client's current health problem is likely attributable to which occurrence?

Superinfection following the eradication of normal oral flora

Diflucan is what class of drug? What is it used to treat?

Systemic Azole Antifungal Treats candidiasis, menginitis, and prophaylxis of candidiasis in bone marrow transplant recipients

A patient has been prescribed oral tetracycline for the treatment of acne. Which of the following must the nurse include in the patient teaching plan?

Take the drug on an empty stomach.

To solve Dilution Problems:

Take the strength of the volume and multiply them together example: 1/4 x 250 mL= 62.5 mL Subtract your volume from this number 250 mL - 62.5 mL = 187.5 mL The number you get tells you how much water to add to the feeding bag, then infuse at the hour given.

Which topical antifungal is drug of choice for kids? What does it treat?

Terbinafine Treats tinea infections

What would the nurse identify as the primary purpose of performing a CD4 count for a patient infected with HIV?

The CD4 count is an indication of the patient's current immunological status.

A doctor orders 500 mL with aminophyllline 0.5 g to infuse at 50 ml /hour. How many milligrams will the client receive each hour?

The IV is 0.5 g or 500 mg in 500 mL. this is equal to 1 mg= 1 mL the patient receives 50 mL per hour so he or she receives 50 mg each hour

Which cells have a slow cell cycle?

The cells of the breasts, testicles, and ovaries

The client has been taking her antibiotic for five days. She tells the nurse that she is now experiencing vaginal itching and discharge. The nurse suspects what has occurred?

The client has developed a superinfection.

A client's health care provider is considering the addition of efavirenz to the client's drug regimen for the treatment of recently diagnosed HIV. Which aspects of the client's medical history should prompt the nurse to question the use of this drug?

The client has hepatitis C and a history of heavy alcohol use.

A 45-year-old client is prescribed acyclovir for the treatment of genital herpes. Which is an expected outcome for this client?

The client will experience fewer recurrences.

You are caring for a patient who has just been diagnosed with a pseudomonas infection. You know that in deciding what medication to order for this patient the physician will take what into account?

The drug that causes the least complication

Penicillins are

The first antibiotic introduced

A sixteen year old client suspected of having HIV/AIDS asks the nurse what causes AIDS. What should the nurse respond?

The human immunodeficiency virus

The nurse is providing health education to a client who has recently been diagnosed with HIV and will soon begin antiretroviral therapy. What teaching point should the nurse prioritize?

The need to adhere rigidly to the prescribed drug schedule

A patient with HIV is taking zidovudine and becomes pregnant. What does the nurse anticipate that the instructions will be for this patient?

The patient will be advised to continue taking the zidovudine.

Infusion pumps

The tubing factor for this is 60 gttt/mL. to solve with an Infusion pump use this formula: number of ML ordered --------------------------- = ml/Hour number of hours to run

A nurse is explaining the rationale for the use of combination therapy in the treatment of HIV infections. Which would the nurse include as the primary reason?

The use of multiple drugs allows attack on the virus at different points in its life cycle.

A client is being treated with saquinavir for AIDS. The client history includes a BMI of 37.98%, hyperlipidemia, and sleep apnea. Based on the client history, why would caution be used when the client is started on saquinavir?

There are potential adverse effects associated with hyperlipidemia.

A client is taking penicillin for an upper respiratory infection. The client calls the office after 2 days of therapy reporting nausea and abdominal pain. Which would be the best instruction for the nurse to give the client?

These are normal side effects, but if they increase in severity or frequency, you need to contact the office again.

Specific antiretrovirals work in different ways. Which describes one mechanism of action of antiretrovirals?

They are protease inhibitors, which block the protease enzyme so the new viral particles cannot mature

How do hormone and hormone modulators work?

They are receptor site specific or hormone specific to block the stimulation of growing cancer cells that are sensitive to the presence of that hormone

Why are antimalarials given in combination?

They attack the parasite at various points in its life cycle

The nurse knows that pseudomembranous colitis is a superinfection of fluoroquinolones, especially when they are administered in high doses, because these medications have what effect in the body?

They disrupt the normal flora of the body.

A protozoan is living in the intestines of your patient. Your patient experiences bloody diarrhea. What would happen to your patient if the parasite dissolves in the tissue of your host?

This could be fatal (your patient has amebiasis)

Which cells are rapidly multiplying cells?

Those of the GI tract, those in hair follicles, skin and bone marrow. Their cell cycle is completed ever few days

Calculating Liquid Injections

To solve liquid injection problems, use the same rule for oral solids and liquids. pick the method you want to use and stick with it. *your method is the formula method*

The nurse is caring for a client taking sulfonamides. The nurse observes sloughing of the skin and mucous membranes. The nurse suspects which hypersensitivity reaction?

Toxic epidermal necrolysis

Indications

Treatment of serious infections caused by susceptible bacteria

What disease is usually spread by men during sexual intercourse who have no signs or symptoms but can cause vaginitis in women?

Trichomoniasis

A client believed he had "pinkeye" and waited several days before seeking care. The health care provider's assessment reveals that the client has a herpes simplex infection in the eye. The nurse should teach the client about what medication?

Trifluridine

True or False: A patient taking metronidazole should avoid alcohol intake.

True (Rationale: Patients take metronidazole should be instructed not to drink any alcohol for at least 3 days after treatment has ended because it can cause severe adverse effects.)

Quinine is used to treat what?

Uncomplicated antimalaria

Fluconazole is excreted where

Urine - excreted unchanged (renal impairment is major concern here)

A client with HIV is frustrated by the size and complexity of the current medication regimen. What should the nurse teach the client about combination therapy for the treatment of HIV?

Use of combination therapy attacks the virus at different stages of the life cycle.

Broad Spectrum of Activity:

Useful in treating a wide variety of infections (ex. p/t with sepsis to blood infection)

What is most effective in treating many cancers?

Using a combination of antineoplastic agents which target different phases of the cell cycle

Recording Intake

Usually when a IVPB is infusing, the PRIMARY stops infusing. After the IVPB is completed, the primary IV flow begins again.

The nursing student asks the instructor why it is more difficult to develop antiviral drugs than anti-infectives. The nursing instructor's best reply would be which?

Viruses are tiny and replicate inside cells.

Are OTC topical preparations used?

Yes. But patients should be cautioned to follow insturction on back and report problems to their physician.

What is something you should monitor your patient for that is taking Amphotericin B (IV) to treat aspergillos?

You should monitor the injection site incase phlebitis occurs

characteristics of common viruses

a virus cannot replicate on its own it must attach to and enter a host cell uses host cells energy to synthesize protein, DNA, RNA viruses are difficult to kill bc they live inside our cells

When infusing micafungin, the nurse will administer it over what period of time? a. 1 hour b. 4 hours c. 24 hours d. 12 hours

a. 1 hour (Rationale: The nurse will administer micafungin over 1 hour.)

A patient is to receive goserelin as treatment for advanced prostatic cancer. The nurse would expect this drug to be ordered for administration by which route? a. Implant b. Oral c. Intramuscular d. Intravenous

a. Implant (Rationale: Goserelin is administered by implant or subcutaneous injection.)

When caring for a patient receiving amphotericin B, the nurse includes what assessment in the plan of care? (Select all that apply.) a. Orientation and reflexes b. Oxygen saturation c. Liver dysfunction d. Renal dysfunction e. Vision

a. Orientation and reflexes c. Liver dysfunction d. Renal dysfunction (Rationale: When administering amphotericin B, it is important to assess the patient's liver and renal function and monitor orientation and reflexes because these are the most vulnerable areas for adverse effects of the drug. Vision and oxygen saturation monitoring may be indicated due to the source of the infection but are not associated with adverse effects from amphotericin B.)

nonnucleoside RTI indications and actions

actions: bind directly to HIV reverse transcriptase blocking both RNA & DNA indications: treatment of HIV 1 infected pt who have experienced clinical or immunological deterioration, in combo with other meds

protease inhibitor actions & indications

actions: block protease activity within HIV virus indications: symptomatic HIV w other meds

CCR5 coreceptor antagonist actions & indications

actions: blocks receptor site on cell membrane to which the HIV virus needs to interact to enter the cell

anti hep b agents

adefovir entecavir telbivudine

Nursing students are studying antiviral drug therapy in a pharmacology class. A student asks the instructor to explain why multi-drug regimens are used to treat HIV infections. The instructor responds that drug combinations are often prescribed because they

are more effective than single-drug therapy.

A patient who is receiving methotrexate is also receiving leucovorin. The nurse understands that leucovorin is given for what reason? a. Synergistic effect on the cancer cells b. Counteract effects of methotrexate c. Reduction of alopecia d. Relief of nausea and vomiting

b. Counteract effects of methotrexate (Rationale: Leucovorin is administered with methotrexate to counteract the toxic effects of methotrexate treatment. Leucovorin does not have a synergistic effect on cancer cells. Leucovorin will not relieve nausea and vomiting or alopecia.)

The nurse reviews the plan of care for a patient receiving antineoplastic therapy and sees a nursing diagnosis of risk for infection. What assessment finding would support this diagnosis? a. Alopecia b. Pancytopenia c. Dehydration d. Gastrointestinal toxicity

b. Pancytopenia (Rationale: Pancytopenia indicates bone marrow suppression, which would reduce the immune response and place the patient at increased risk for infection. Gastrointestinal toxicity might suggest a nursing diagnosis of imbalanced nutrition, diarrhea, or risk for dysfunctional gastrointestinal motility. Dehydration might suggest a nursing diagnosis of deficient fluid volume. Alopecia may suggest a nursing diagnosis of disturbed body image.)

A 49-year-old client is diagnosed with TB. The client has a history of alcoholism but has been sober for 3 months now. The client has been prescribed INH. What should the nurse ensure is obtained before initiating the therapy?

baseline liver function test values

Antifungals can be administered in pregnant women when the ______ outweighs the risk

benefit

An immunosuppressed patient develops pneumocystis carinii pneumonia (PCP) and is to receive atovaquone as treatment. The nurse would instruct the patient to continue this therapy for how long? a. 7 days b. 14 days c. 12 days d. 5 days

c. 12 days (Rationale: Atovaquone for treatment of PCP is used for 12 days.)

A patient is receiving carboplatin. The nurse would expect to administer this drug by which route? a. Oral b. Intramuscular c. Intravenous d. Subcutaneous

c. Intravenous (Rationale: Carboplatin is administered IV on day 1 every 4 weeks.)

A nurse is preparing to administer primaquine therapy to a patient. The nurse would anticipate administering the drug by which route? a. Inhalation b. Intramuscular c. Oral d. Subcutaneous

c. Oral (Rationale: Primaquine, like all the antimalarial agents, is administered orally.)

After teaching a group of students about topical antifungal agents, the nursing instructor determines that the students have understood the material when they identify what correct information about these drugs? a. They can cause significant drug-drug interactions. b. The drugs can be used for an indefinite period of time. c. The drugs are often too toxic for systemic administration. d. Application site reactions rarely occur with topical application.

c. The drugs are often too toxic for systemic administration. (Rationale: The antifungal agents reserved for topical use are often too toxic to be given systemically. Systemic antifungals are associated with numerous drug-drug interactions. Topical agents are not absorbed systemically so they do not have drug-drug interactions. Most of the topical agents are used for a specific period of time, usually no longer than 4 to 6 weeks depending on the drug.)

A patient who is receiving primaquine comes to the clinic with complaints of nausea, vomiting, and vertigo. What other finding would the nurse expect to assess if the patient was experiencing cinchonism? a. Rash b. Fever c. Tinnitus d. Dyspepsia

c. Tinnitus (Rationale: Cinchonism, due to high levels of primaquine, is manifested by nausea, vomiting, vertigo, and tinnitus. Fever, dyspepsia, and rash are adverse effects associated with antimalarial therapy; they are not associated with cinchonism.)

20 mL syringe

calibrated in 1 mL incraments

A client prescribed rifaximin for diarrhea has developed frank bleeding in the stool. What intervention should the nurse anticipate being implemented to best ensure client safety?

changing to a different antibiotic

IV push medications

check the compatibility of medication and IV solution, typically given over 1-5 minutes

Fungi differ from bacteria because they have _________ in their cell walls and _________ in their cell membranes

chitin ergosterol

Rimantadine (flumadine)

class: agents for influenza A indications: prophylaxis and treatment of illness caused by influenza A virus adverse effects: light headedness, dizzy, insomnia, nausea, dyspnea, orthostatic hypotension, depression

simeprevir (olysio)

class: anti hep c agents indications: chronic hep c in adults with compensated liver dysfunction in combo with peginterferon and ribavirin actions: inhibits hep c protease formation preventing viral replication adverse effects: fatigue, nausea, diarrhea, rash

In which clients are the use of cidofovir contraindicated?

clients with renal impairment

A patient with giardiasis is to receive tinidazole. The nurse would instruct the patient to do what? a. "Take the drug twice a day for 12 days." b. "Take the drug each morning on arising for 5 days." c. "Take the drug for 3 consecutive days." d. "Take the drug as a single dose with food."

d. "Take the drug as a single dose with food." (Rationale: For giardiasis, tinidazole is taken as a single dose with food. Taking the drug for 3 consecutive days would be appropriate for the patient with amebiasis. Metronidazole would be used for 5 to 10 days. Twice-daily dosing for 12 days would be appropriate for atovaquone as treatment for PCP.)

A patient is to receive doxorubicin as treatment for leukemia. The nurse would instruct the patient that the typical cycle would be repeated at which frequency? a. Every 4 weeks b. Every other week c. Every week d. Every 3 weeks

d. Every 3 weeks (Rationale: Doxorubicin is administered as a single IV dose and repeated every 21 days [3 weeks]).

integrase inhibitors drug drug i

decreased serum level of either drug if combined with rifampin

nonNRTI drug drug i

delavirdine, dapsone, antiTB drugs, CA channel blockers, warfain, quinidine, indinavir, efavirenz, midazolam, rifabutin, triazolam, ergot derivatives

Sites for Intramuscular Injections

deltoid muscle (up to 1 mL), vastus lateralis, rectus femoris, dorsogluteal, and ventrogluteal muscle

nonnucleoside reverse transcriptase inhibitors

delvirdine efavirenz nevirapine rilpivirine

There are four different ways to solve these types of problems, what are they?

dimensional analysis formula method ratio proportion *Formula works best for me!* When using a formula method with oral solids, the supply is typically either 1 tablet or 1 capsule.

integrase inhibitors

dolutegravir raltegravir

oral liquids

elixirs, suspensions, emulsions, syrups, extracts

A nurse is providing care to a client who is receiving a protease inhibitor with a nursing diagnosis of Disturbed Body Image related to a redistribution of fat to the abdomen. Which intervention would the nurse include in the client's plan of care?

encouraging the client to verbalize feelings about body changes

A nurse provides medication teaching related to the importance of adhering to the saquinavir drug regimen. In order to best minimize the risk of increased fatty-like tissue, the nurse will instruct the patient to

engage in regular exercise

Example: Order: Amoxil 1 g Supply: 1 tablet Have: 500 mg

equivalent is : 1 g = 1000 mg so 1000mg / 500 mg x 1 tablet = 2 tablets

Macrolide drugs include

erythromycin (Ery-tab, Eryc, others)

Angiogenesis

formation of new blood vessels

integrase inhibitors adverse effects

headache dizzy increased risk for dev of rhabdomyolysis and myopathy

anti hep b agents adverse effects

headache dizzy nausea diarrhea increased liver enzymes

A nurse, reviewing a newly admitted client's previous health records, notes that the client is currently prescribed acyclovir. The nurse should assess the client for signs and symptoms associated with what medical diagnosis?

herpes simplex virus

When assisting the patient to interpret a negative HIV test result, the nurse informs the patient that the results mean

his body has not produced antibodies to the AIDS virus

hep b agents drug drug i

increased risk renal toxicity if drugs are taken with other nephrotoxic drugs

CCR5 co. antagonist drug drug i

increased serum level and toxicity when combined with cytochrome P450 CYP3A

In children, dangers of __________ are much worse than potential ______ of medication

infection; risks

viruses that respond to antiviral therapy

influenza A, some resp viruses, herpes virus, cytomegaiovirus, HIV that causes AIDS, hep B & C, some viruses that cause warts & certain eye infections

Bactericidal -

kill the cell

right drug

label and compare to MAR, strength, allergies, if unsure look it up in the drug book, expiration date

LR

lactated ringers solution

long acting insulin

lantus, begins working in 2-4 hours, lasts 24 hours, clear and can not be mixed with other insulins

Rapid acting insulin

lispro, begins working in 5-15 minuets, clear

Many of the azoles are associated with ________ toxicity and can cause severe effects on a fetus or nursing baby

liver

Anaplasia

loss of organization and structure; property of cancer cells

fusion inhibitor contraindications

lung disease pregnancy

agents for herpes & cyto adverse effects

nausea, vom, headache, rash, hair loss, paresthesias, neuropathy, renal dysfunction

agents for herpes & cyto drug drug i

nephrotoxic drugs zidovudine

Can you half or divide EC (enteric coated medication)

no never

protease inhibitor drug drug i

nonsedating antihistamines sedatives/hypnotics antiarrhythmics

Order: Amoxil oral susp 500 mg Supply: 125 mg Have: 5 mL

only give 20 mL 500 mg / 125 mg x 5 mL= 20 mL

order: 0.75 g Supply: 250 mg Have: 1 mL

only give 3 mL

Order: Coreg 6.25 mg PO bid Supply: 1 Tablet Have: 12.5 mg

only give a 1/2 of a tablet 6.25mg / 12.5 mg x 1 Tablet = 1/2

A young adult, diagnosed with hepatitis B virus (HBV), has been prescribed treatment with lamivudine. Shortly after beginning treatment, the client reports experiencing intense abdominal pain. The nurse should recognize the possibility of what medication-associated adverse effect?

pancreatitis

A client has been admitted with leprosy. The nurse completes the health history and physical examination. In addition to a thorough assessment of the skin, what other areas of the body would be important to assess? (Select all that apply.)

peripheral nerves mucosa of the gastrointestinal tract

Infusion sets

plastic tubing attached at one end to the IV bag and at the other end to a needle or catheter inserted into a blood vessel

NRTI contraindications

pregnancy hepatic/renal bone marrow suppression

Chloroquine is used to treat and ________ malaria

prevent

Bacteriostatic -

prevent reproduction of the cell

agents for influenza A drug drug i

primarily anticholinergic agents

Adults should be advised about ________ treatment for protozoal infections.

prophylaxis treatment

The nurse is reviewing the medication history of a client who is prescribed penicillin therapy. Use of which medication would alert the nurse to the possibility of the client's increased risk for an anaphylactic reaction?

propranolol

A nurse is administering a drug that blocks the protease enzyme so new viral particles cannot mature. Which category of drugs is the nurse most likely administering?

protease inhibitors

Short acting insulin

regular insulin, begins working in 30 minuets, clear

Antiviral drugs should be used during pregnancy only when the benefit outweighs the risk to the fetus. Which antiviral drug would the nurse identify as being absolutely contraindicated during pregnancy?

ribavirin

Antituberculosis drugs include

rifampin (Rifadin) (1st line drug)

microdrip

small needle in the chamber 60gtt/mL

Antiinfectives act on what?

specific enzyme system or biological process, many microorganisms that do not act on this system are not affected by this particular drug

A 7-year-old child has been taking tetracycline for a bacterial infection. The nurse will be sure to inform the parents that this drug could cause

staining of permanent teeth.

Topical antifungals are not absorbed __________

systemically

oral medication

tablets, capsules, powders, oral liquids

Tetracyclines should not be used in children younger than 8 years of age because of their effects on:

teeth and bones.

When is it okay to apply a topical cream/lotion?

when the area has been cleaned and dried

Candidiasis is caused by

yeast (thrush)

Antiviral therapy for influenza is being administered within your long-term care facility; it commenced about 3 weeks ago. The nurse would monitor for which adverse effects? (Select all that apply.)

• Anorexia • Nausea

To reproduce, a virus needs the cellular material of another living cell (the host cell), which of the following are steps in the viral replication process? (Select all that apply.)

• Attachment of the virus to host cell • Release of viral DNA and RNA inside the host cell • Replication of the viral DNA by the host cell • Death of the host cell

What should the nurse's assessment of the client prior to administration of antiviral drugs include? (Select all that apply.)

• client's general state of health • resistance to infection • vital signs

An HIV-positive patient comes to the clinic and asks the nurse if the patient should receive a flu vaccine. The patient states, "I never get the flu, so why should I take it?" What is the best response by the nurse?

"All HIV-positive patients should receive the flu vaccine as prophylaxis."

A client comes to the emergency department reporting a throbbing headache, nausea, vomiting, chest pain, dyspnea, vertigo, and blurred vision. The client reveals taking cefaclor for an infection. Which question would the nurse ask next?

"Have you had any alcohol to drink in the past 72 hours?"

Aminoglycoside drugs include

"micins" gentamicin, streptomycin

Use microdrip when:

* IV is administered for a long time *a small amount is being infused *the macrodrops are to few

Use Macrodrop when:

*the order specifies a large amount over a shorter period of time *the microdrips per min are too many, and counting the drip rates becomes to difficult.

Contraindications and cautions for cephalosporins are:

- Avoid use on patient with allergy to cephalosporins or penicillins - Caution on patients with hepatic and renal impairment - Caution with pregnancy or lactation (effects on infant known)

Aminoglycoside adverse effects:

- CNS effects: ototoxicity (deafness), vestibular paralysis, confusion, depression, disorientation, numbness, tingling and weakness - Nephrotoxicity (renal toxicity): renal failure and bone marrow depression - GI effects: nausea, vomiting, diarrhea, weight loss, stomatitis, hepatic toxicity - Cardiac effects: palpitations, hypotension, hypertension - Hypersensitivity: purpura, rash, urticaria, exfoliative dermatitis

Drug-food interactions for macrolides include:

- Food in stomach decreases absorption of oral macrolides - Should be taken on an empty stomach with a full glass of water, 1 hour before or 2-3 hours after meal

Indications for fluroquinolones are:

- Indicated for susceptible strains of gram-negative bacteria - Cipro is good for broad spectrum or gram-negative bacteria - Good for: anthrax infection, typhoid fever, otitis externa

Adverse effects of tetracyclines:

- Irritation to GI tract - Fatal hepatotoxicity - Skeletal effects: bone and teeth damage - Dermatological effects: photosensitivity and rash - Superinfections - Topical or ocular application: pain and burning - Hematological (least common): hemolytic anemia and bone marrow depression - Hypersensitivity: urticaria to anaphylaxis, intracranial hypertension

Contraindications and caution for tetracyclines include:

- Known allergy to tetracyclines or tartrazine - Cannot be taken during pregnancy or lactation (effects on bone and teeth) - Ophthalmic preparation is contraindicated on patients who have fungal, mycobacterial, or viral ocular infections - Caution with children under 8 years old (bone and teeth development) - Caution with hepatic and renal dysfunction

Patient teaching for macrolides:

- Take safety precautions if CNS effects occur - Drink lots of fluids and maintain nutrition - Report difficulty breathing, severe headache, diarrhea, skin rash, and mouth or vaginal sores

Patient teaching for tetracyclines:

- Take throughout day for best results - Take on empty stomach, 1 hour before or 2-3 hours after meals, with full glass of water - Do not take with food, diary products, iron preparations, or antacids - Take full course - Do not save medication, outdated can be toxic to kidneys - Superinfections in your mouth or vagina can occur - Keep out of reach of children - Use sunscreen and protective clothing if sensitivity occur - Report changes in color of urine or stool, severe cramps, difficulty breathing, rash or itching, or yellowing of skin or eyes

Nursing Considerations-Assess

-Allergy to aminoglycoside -Perform Physical assessment -Perform C&S test at infection site -Assess vital signs -Perform renal and liver function test

When giving intramuscular injections:

-Always aspirate before injecting -If blood is returned, discard the dose

What is Bacitracin?

-An antibiotic that interferes with the cell wall synthesis of susceptible staphylococcal bacteria. -Adverse effects include nephrotoxicity and superinfection. Because of the development of resistant strains and more potent antibiotics, bacitracin is now indicated only for the treatment of respiratory infections in infants caused by susceptible staphylococci, treatment of eye infections, prevention of infections in minor skin wounds, and treatment of minor skin infections caused by susceptible strains of staphylococci. -Bacitracin is available in intramuscular, ophthalmic, and topical preparations.

Antiinfective Activity:

-Antiinfectives vary in their effectiveness against invading organisms. -Some are selective -Bactericidal -Bacteriostatic

Therapeutic Actions of Antiinfective Agents:

-Changes cell wall function -Interfere with biosynthesis of the bacterial cell wall -Prevent the cells of the invading organism from using substances essential to their growth and development -Interfere with steps involved in protein synthesis -Interfere with DNA synthesis -Alter the permeability of the cell membrane to allow essential cellular components to leak out

Human Immune Response:

-Goal of antiinfective therapy is reduction of the population of the invading organism. -Drugs that would eliminate all traces of any invading pathogen might be toxic to the host as well. -Immune response is a complex process involving chemical mediators, leukocytes, lymphocytes, antibodies, and locally released enzymes and chemicals.

Which of the following has been found to be as important as drug therapy when treating infections in the immunosuppressed?

-Good hand washing -Maintaining standard precautions -Proper nutrition

Treatment of Systemic Infections:

-Identification of the infecting pathogen is done by culture -Sensitivity testing to determine which drugs are capable of controlling the particular microorganism -Combination therapy

Adverse Reactions to Antiinfective Therapy:

-Kidney Damage (renal) -GI Toxicity (nausea, vomiting, liver damage) -Neurotoxicity (nerve cells) -Hypersensitivity Reactions (anaphylaxis-hives) -Superinfections (destruction of normal bacteria-yeast infection, CDIF)

Preventing Resistance:

-Limit the use of antimicrobial agents to the treatment of specific pathogens sensitive to the drug being used -Make sure doses are high enough, and the duration of drug therapy long enough -Be cautious about the indiscriminate use of antiinfectives

Resistance:

-Natural or acquired -Antiinfectives act on specific enzyme system or biological process, many microorganisms that do not act on this system are not affected by this particular drug

1920s:

-Paul Ehrlich worked on developing a synthetic chemical effective only against infection-causing cells -Scientists discovered penicillin in a mold sample

Pharmacokinetics

-Poorly absorbed from the GI tract; rapidly absorbed after IM injection, reaching peak after 1 hour -Widely distributed throughout the body; crossing placenta and entering breast milk -Excreted unchanged in the urine; half life of 2 to 3 hours -Depends on kidney for excretion; toxic to kidney

Acquiring Resistance:

-Producing an enzyme that deactivates the antimicrobial drug -Changing cellular permeability to prevent the drug from entering the cell -Altering transport systems to exclude the drug from active transport into the cell -Altering binding sites on the membranes or ribosomes, which then no longer accept the drug -Producing a chemical that acts as an antagonist to the drug

Combination therapy:

-Use of a smaller dosage of each drug -Some drugs are synergistic -In infections caused by more than one organism, each pathogen may react to a different anti-infective agent -Sometimes, the combined effects of the different drugs delay the emergence of resistant strains

IV piggyback

-secondary volume (mL) -secondary rate (mL/hr)

Use microdrip when

-the IV is to be administered over a long period -a small amount of fluid is to be infused -the macrodrops per minute are too few

Use macrodrop when

-the order specifies a large amount of fluid over a short time -the microdrips per minute are too many, and counting the drip rate becomes difficult

Need to know to program the pumpo

-volume for infusion (mL) -rate for infusion (mL/hr)

0.33% NS

0.33% Normal Saline in water (1 /3 normal saline)

0.45% NS

0.45% Normal Saline in water (1 / 2 normal saline)

0.45% NS

0.45% saline in water (1/2 normal saline)

Volume of intramuscular injection

0.5 mL - 3 mL more than 3 mL should be divided

0.9% NS

0.9% Normal Saline in water (normal saline)

0.9% NS

0.9% saline in water (sometimes termed normal saline)

1000 mL

1 L

1 qt=

1 L or 1000 mL

2.54 cm

1 in

Maximum amount for subcutaneous

1 mL can be injected in an ADULT

maximum injection for 0-5 years old IM

1 mL per injection

typical needle for intradermal

1 mL syringe 25-27 gauge 1/4" - 5/8" length

typical needle for subcutaneous

1 mL syringe 25-27 gauge 3/8" - 5/8"

30 mL

1 oz

500mL=

1 pt= 2 cups= 16oz

1000ml=1L=

1 qt= 2 pt

15mL = _tbsp

1 tbsp= 1/2 fl oz

5 ml

1 tsp

What are the ways a tumor grows?

1. Primary tumor grows and invades surrounding tissue 2. It moves into the basement membrane of capillaries surrounding the tissue 3. It enters the bloodstream 4. It leaves the bloodstream through capillary walls 5. It proliferates at this new site, grows, and invades surrounding tissue

Steps for Reconstituting powders with directions

1. Read the order 2. Identify the supply and directions for dilution 3. Dilute the fluid 4. Identify the solution and the new supply 5. Calculate the amount to give and prepare the amount 6. Write on the label: sol made date time your initals exp. date 7. Store according to directions

Steps to mixing insulin

1. clean tops of vials 2. draw up total amount of insulin in air 3.air in cloudy(NPH) 4.air in clear(regular) 5. draw out of clear(regular) 6. draw out of cloudy(NPH).

Order: 500 mg Supply: 250 mg Have: 5 mL

10 mL

Vaginal suppositories should be inserted high into vagina with patient remaing recumbent for how long?

10-15 minutes

1 liter

1000 mL

1 milligram

1000 mcg (microgram)

1 gram

1000 mg

An IV of 1,000 mL is infusing at 10 microdrips per min. what is the intake for 8 hours?

10000 x 10 -------------- = 21 gtt/min 480

Baxter-Travenol macrodrip

10gtt/mL

Example: The primary IV is infusing at 50 mL/hour. For 12 hours, the IV intake would be 600 mL (12 x 50 = 600). If you administer an IVPB of 100 mL once during the 12 hours and infuse the IVPB at 100 ml /hour, then your intake would be as follows:

11 hours x 50 of primary IV = 550 1 HOUR of IVPB x 100 = 100 --------------------------------------- 12 hours= 650 mL intake.

120mL of D5W over 60 minutes with a drop factor of 10 drops per mililiter

120X10/60=20gtt/minute

120mL of IV fluid over 60 minutes

120mL/1hour=120mL/hour

Order: 0.35 g Supply: 125 mg Have: 5 mL

14 mL

1 mL =

15 gtt (drops)

1 tbsp=____ mL

15 mL

Abbott

15gtt/mL

administering insulin

2 licensed nurses check by independent verification

maximum injection for 6-12 years old IM

2 mL per injection

Pregnancy should be avoided during therapy and for how long after therapy?

2 months

30mL=

2 tbsp = 1 fl oz

1 kg

2.2 lbs

1 kg = ? lbs

2.2 lbs

1 inch = ? cm

2.54 cm

Order: 1 g in 250 mL IVPB 7am Supply: 500 mg powder reconstitute with 10 mL sterile water to yield 50 mg/ mL Tubing factor: 10 gtt/ mL Package insert directions: 250 mL D5W Run over 2 hours

21 gtt/ min 250 ml x 10 --------------- = 21 gtt 120 min

A client is caring for an 8-year-old child who weighs 30 kg. The health care provider orders gentamicin IM for the client. The recommended dosage range is 2-2.5 mg/kg q8h. What is the maximum amount of gentamicin the client will receive in a day?

225 mg

A patient is receiving an antibiotic IVPB in 250 mL q6h. What is the 24 hour parenterally intake?

250 x 4 = 1000 (24 / 6=4, 4 times 250 mL was infused in 24 hours) Answer is 1000 mL

250mL D5W IV at 250mL/hr

250mL to infuse over 1 hour

90 mL

3 oz

how many times do you check 6 rights?

3 times during administration: 1) during preparation against the MAR 2) After preparation 3) bed side, also check BP/pulse if needed

1 oz (or 1 fl oz)= ____ mL

30 mL

1 gallon

4 qt

Levofloxacin 750 mg IV is ordered for a client with a urinary tract infection. The medication is to mixed yielding 250 mg/15 mL. How many mL should be drawn up in the syringe?

45

You should administer the entire course of drug to get the full beneficial effects. This may take as long as _______________

6 months

To deliver 1mL of fluid to the client (all microdrips)

60 drops drip in the drip chamber (60gtt=1mL)

Order: Lasix 60 mg by mouth everyday Supply: 1 tablet Have: 40 mg per tablet

60/40 x 1 = 1 and a half tablets only give 1 and a half tablets

Tubing factor for an IV infusion pump

60gtt/mL

A patient/ client is receiving an antibiotic IVPB in 75 mL q6h to run over 1 hour plus a maintenance IV of 125 mL/hour. What is the 24 hour intake parenterally?

75 x 4 = 300 mL (24 hours/ 6 hours is 4 times 75 mL is being infused over 24 hours) 125 x 20 = 2500 mL answer is 2800mL

240 mL

8 oz

An Iv of 1000 mL is infusing at 10 microdrips per min. what is the parenterally intake for 8 hours?

80 mL 10 microdrips x 8 = 80 mL

Example: Order: 1000 mL D5W IV 8AM-8PM Available: an infusion pump How long is it running?: 12 hours

83 mL/hr 1000 mL ----------- = 83.333 rounds to just 83 (round to whole#) 12 hours

1 cup =

8oz= 240-250mL

A total of 900 mL of an IV solution is to infuse at 100 mL/ hour. If it is 9 am when the infusion starts, at what time will it be completed?

900mL ----------- = 9 HOURS 100 ml 9 AM + 9 HOURS = 6 PM

For which clients would treatment with acyclovir be most clearly indicated?

A 77-year-old whose recent debilitating pain has been attributed to a herpes zoster infection

Extreme caution would be necessary with the use of gentamicin in which client?

A client who has chronic renal failure secondary to diabetes mellitus

Natural or acquired:

Ability over time to adapt to an anti-infective drug and produce cells that are no longer affected by a particular drug.

Metastasis

Ability to enter circulatory or lymphatic system

Example: Order: 1000 mL D5W with 20 mEq KCl IV 10 am- 10 pm Available: a vial of KCl 40 mEq/20 mL; Micro 60gtt/min, macro 20 gttt/min How long it's running?: 12 hours or 600 mintues Calculate how many mL to add to the IV Fluids first, then solve both tubing's and pick which one you want to use.

Add 10mL to the Iv bag 20 mEq ------------ x 20 mL = 10 mL 40mEq Micro: 1000 mL x 60 ------------------ = 83 gtt/min 720 min Macro: 1000 mL x 20 ------------------ = 28 gtt/min 720 min

Example: Prepare 1 / 2 of Isocal at 60 mL/hour. Total volume will equal 250 mL

Add 125 mL of water 1 /2 x 250 = 125 250 - 125= 125 mL

Prepare 3 /4 Isocal at 6o mL/ Hour. Total volume will equal 250 mL

Add 62.5 3 /4 x 250= 187.5 250 - 187.5 = 62.5 mL

Trypanosomiasis can cause what two disease

African sleeping sickness Chagas disease (passed by a housefly)

A nurse is assessing a 66-year-old man who is HIV-positive. The patient has been prescribed didanosine (Videx). It would be most important to question the patient about which of the following?

Alcohol use

How is malaria transmitted?

Anopheles mosquito

A client is receiving a nonnucleoside reverse transcriptase inhibitor. Why should the nurse caution the client to avoid the concurrent use of St. John's wort?

Antiviral effects may be reduced.

Which statement regarding antiviral medications is true?

Antivirals do not eliminate existing viruses from tissues.

If a patient was prescribed Amphotericin B, what other drug should they NOT be prescribed

Any drug that is known to cause nephrotoxicity (Corticosteroids, Aminoglycosides)

What can the nurse do to maximize a patient's adherence to the medication regimen?

As an advocate of the patient, discuss benefits and risks of antiretroviral therapy as well as administration.

What should you advise your patient to avoid while taking Metronidazole?

Avoid drinking Alcohol for at least 3 days after

A patient with AIDS is being prepared for discharge. The nurse caring for the patient with AIDS knows the patient receives Agenerase. What dietary counseling will the nurse provide based upon the patient's medication regimen?

Avoid high-fat meals while taking this medication

A patient with TB has been admitted to a health care facility. When providing instructions for the patient teaching related to antitubercular drugs, which instructions should the nurse provide in order to avoid complications in the patient's GI tract?

Avoid the consumption of alcohol.

The ability of bacteria to produce substances that inactivate or destroy the antibiotic is known as which of the following?

Bacterial resistance

An older adult client, diagnosed with community-acquired pneumonia, has been prescribed aztreonam. What action should the nurse perform before administering the first dose?

Confirm the client's allergy status.

Clotrimazole is given OTC for treatment. What are the side effects if given as a cream? What are the side effects if given as a troche? What about if a suppository in the vagina?

Cream: burning, itching, rash, swelling, irritation Troche: nausea, vomitting, abnormal liver function Vaginal: lower abdominal pain, urinary frequency, burning or irritation in sexual partner

What should the pre-administration assessment of any antitubercular drug include?(Select all that apply.)

Culture and sensitivity testing Complete blood count Family and contacts history Radiographic studies Medication history

What do antimetabolites inhibit?

DNA production in cells that depend on certain natural metabolites to produce their DNA

What are major signs and symptoms of malaria?

Destruction of blood cells and toxicity

Tetracyclines were

Developed as semisynthetic antibiotics and composed of four rings

D

Dextose

A client is taking clindamycin for an infectious process and presents to the health care provider's office with symptoms of dehydration. The nurse understands that the client is experiencing what adverse reaction from the administration of clindamycin?

Diarrhea

Your patient has been taking Chloroquinine to treat malaria. They are no longer able to see. What would a proper nursing diagnosis be for this patient?

Distubed sensory perception (visual) related to CNS effects

A nurse is caring for a client diagnosed with a serious gram-negative infection prescribed amikacin. Which action should the nurse prioritize while caring for this client?

Do renal tests daily

Patient teaching

Drink lots of fluids, maintain nutrition (very important) vomiting and diarrhea may occur

REMEMBER:

Drip rates are always rounded to the nearest whole number unless using an infusion pump that can infuse in tenths or hundredths.

True or False: Cancer cells can rapidly develop a resistance to alkylating agents.

False (Rationale: Neoplastic cells can rapidly develop resistance to antimetabolites.)

True or False: No cancer cells have been identified that can remain dormant for more than 2 years.

False (Rationale: No cells have been identified that can remain dormant for more than 5 years.)

True or False: Terbinafine is a better choice for patients who need to take medications that are metabolized by the CYP 2D6 enzyme system.

False (Rationale: Terbinafine inhibits the CYP 2D6 enzyme system, so it may be a better choice for patients who require drugs that are metabolized by the CYP 450 system.)

Your patient has an amebic infection. What should you prescribe them?

Flagyl (metronidazole)

What are the four Systemic Antifungals that have a pediatric dose and are safe to administer in children?

Fluconazole (Azole systemic) Ketoconazole (Azole systemic) Terbinafine (topical) Griseofulvin (other antibiotic)

Which antifungals are more common in causing renal impairment?

Flucytosine Fluconazole Griseofulvin

Oral Liquids

For Liquids you can pick the same method to solve except the supply will include a liquid measurement in mL.

After reviewing information about the drugs used to treat herpes virus and CMV infections, a group of students demonstrate understanding about these drugs when they identify which of the following as being available only for intravenous administration?

Foscarnet

FOR BG LESS THAN 80

GIVE 1 AMP OF D50 AND CALL THE DOCTOR

FOR BG 101-150

GIVE 2 UNITS OF REGUALR INSULIN

FOR BG 151-200

GIVE 3 UNITS OF REGULAR INSULIN

FOR BG 201-250

GIVE 5 UNITS OF REGUALR INSULIN

FOR BG 81-100

GIVE NO UNITS AT ALL

FOR BG GREATER THAN 150

GIVE THE NUMBER OF UNITS OF REGUALR INSULIN BASED ON: BG-100/40 (Blood glucose minus 100 divided by 40 ) in this formula the denominator may change depending on the order and how controlled the patients blood sugar needs to be. a lower number in the denominator such as BG-100/30, increases the amount of insulin that the patient receives.

Common meds

Gentamycin, Kanamycin sulfate, Neomycin sulate, Steptomycin Sulfate

Which topical antifungal has the ability fight thursh but has been known to cause a purple stain in a child's mouth?

Gentian Violet

Your patient experiences rotten egg smelling stool, and pale and mucus filled stool. What protozoan infection may they have? What else may they experience due to this type of sign?

Giardiasis The extreme diarrhea could cause anorexia, malnutrition, etc.

Order: 1,500 units Supply: 1 mL Have: 5,000 units Use a 3 ml syringe

Give .3 mL 1500 units / 5000 units x 1 mL = 0.3 mL the answer is already in the tenths place

Order: 120 mcg Supply: 1 mL Have: 250 mcg Use a 1 mL syringe

Give 0.48 mL IV. 120 mcg / 250 mcg x 1 mL = 0.48 Answer must be in the thousandths, and it already is . No rounding needed.

Order: 4 mg IM Supply: 1 mL Have: 8 mg Use a 3 mL syringe

Give 0.5 mL IM 4 mg / 8 mg x 1 mL = 0.5 the answer is already in the tenths place.

Order: 1.5 mg IM Supply: 1 mL (For liquid calculations, the supply is usually 1 mL although there are exceptions ) Have: 2 mg Use a 3 mL syringe

Give 0.8 mL IM 1.5 mg / 2 mg x 1 ml = 0.75mL since we are using a 3 mL syringe we have to round to the nearest tenth so your answer should be in tenths: 0.8 mL.

Order: 1 mg Supply: ampule labeled 1:1000 (1 mg/ mL) Equivalent: 1:1000 means 1 g per 1000 mL 1 g= 1000 mg, Therefore 1000 mL contains 1000 mg and 1 mL contains 1 mg. So your supply is really 1000 mL Have: 1000 mg in 1000 mL

Give 1 mL 1 mg / 1000 mg x 1000 mL = 1

Order: 0,2 mg Supply: 1: 5000 1 g per 5000 mL 1 g = 1000 mg Therefore 5000 mL contains 1000 mg So your supply is 5,000 mL Have: 1000 mg in 5,000 mL

Give 1 mL 0.2 mg / 1000 mg x 5000 mL= 1

Order: 1 million units Supply: 5 million unit vial Fluid: use sterile water sol and new supply: 1 million units Have: 1 million units in 1 mL

Give 1 mL 1 million units / 1 million units x 1 mL = 1 mL

Order: 0.3 g Supply: 500 mg powder Fluid: 2.0 mL sterile water Sol and new supply: 225 mg/ mL Have: 225 mg/mL

Give 1.3 mL 300 mg / 225 mg x 1 ml = 1.33 or 1.3 mL

Order: 30 mg Supply: 2% 2 g per 100 mL 1 g = 1000 mg 2 g = 2000 mg Therefore 100 ml contains 2000 mg so your supply is 100 mL Have: 2000 mg

Give 1.5 mL 30 mg / 2000 mg x 100 mL = 1.5

Order: 100 mg Have: 125 mg Supply: 2 mL

Give 1.6 mL 100 mg / 125 mg x 2 mL = 1.6

Example Order: 0.65 g Supply: The fluid portion of the solution made . In this example it is 1 mL = 325 mg Have: The strength of the drug is applied. The example 1 g as a dry powder, when reconstituted is 325 mg/mL. Remember that the manufacturer gives you the strength of the solution. You do not have to determine it.

Give 2 mL 650 mg / 325 mg x 1 mL = 2 mL Store according to the directions. Label the vial with the soluntion made date time initials exp. date and initial your initials so: 325mg/mL 10/16/12 1600 (4pm) BJJ Expires 10/19/12

Example: Order: 0.5 g Supply : 1 g vial of powder Have: 250 mg/mL

Give 2 mL 500 mg / 250 mg x 1 mL = 2 mL

Order: 1 g Supply: 50% 50 g per 100 ml your supply is 100 mL Have: 50 g

Give 2 mL 1 g / 50 g x 100 mL = 2 mL

Order: 20 mg Have: 10 mg Supply: 1 mL

Give 2 mL IV 20 mg / 10 mg x 1 mL = 2

Who is more likely to experience a hemolytic crisis while taking Chloroquine?

Greeks, Italians, Mediterranean descent, and African American males

Which drug is given orally to treat tinea infections?

Griseofulvin (drug of choice for kids) can cause liver and hepatic toxicity

A client with active AIDS is infected with a retrovirus. This virus is better known as which?

HIV

IV Medications

IVP medications must be diluted and administered either according to directions in a nursing drug book or according to hospital policy. First calculate the correct dose.

Which of the following would be least likely to cause a drug interaction when rimantadine is prescribed?

Ibuprofen

Systemic fungal infections are increasing with the rise of ____________ patients

Immunocompromised

A female client is diagnosed with hepatitis C as well as a urinary tract infection. The organism is sensitive to tetracycline. Why is the health care provider reluctant to order tetracycline in a client with hepatic impairment?

It slows drug elimination.

Contraindications

Known allergies, renal or hepatic disease, hearing loss

Which of the following medications is contraindicated in children?

Levofloxacin

A patient with chronic hepatitis B (HBV) infection is scheduled to begin a new treatment regimen that will include adefovir dipivoxil (Hepsera). What assessments should be prioritized before the initiation of this drug treatment?

Liver function testing and HIV testing

Which would be most important to stress with a client who is receiving adefovir for treatment of chronic hepatitis B?

Maintenance of a continuous adequate supply of drug

Order: 500 mL NS iv 12 Noon- 4pm Available: microdrip at 60 gttt/mL; macrodrip at 20 gttt/mL How long will the IV run? 4 hours Because there us no pump available, the nurse must choose the drip factor. Solve for both drip factors and choose one. Round to the nearest whole number

Micro: 500 mL x 60 gttt ------------------- = 125 gttt/min 240 min (60 x 4) Macro: 500 mL x 20 gttt ------------------ = 41. 6 or 42 gttt/ min 240 min Choose either one label the IV set the drip rate

Order: 500 mL D5W IV KVO for 24 hours Available: microdrip at 60 gttt/min; macro at 10 gttt/min How long will it run? 24 hours or 1440 minutes Work out the problem and figure out which tubing is best to use for this patient.

Micro: 500 ml x 60 gttt ------------------ = 21 gttt/ min 1440 min Macro: 500 mL x 10 gttt --------------------- = 3 gtt/min 1440 min Choose the Micro why? it;s being administered for a long period of time a small amount is being infused the macrodrips are to few

In anticipation of a prescription for delavirdine mesylate, what instructions should the nurse provide to the client?

Mix the drug with water and drink the resulting solution.

A 75-year-old patient with a history of renal impairment is admitted to the primary health care center with a UTI and has been prescribed a cephalosporin. Which of the following interventions is most important for the nurse to perform when caring for this patient?

Monitoring blood creatinine levels.

Intermediate acting insulin

NPH, begins working in 2-3 hours and lasts 14-24 hours, cloudy, can be mixed with regular insulin

halving or dividing medications that are sustained release or long-acting is acceptable.

No!!

Is it okay to scratch an area being treated with antifungal medication?

No. use cool compresses to decrease itching

agents for HIV & AIDS

Nonnucleoside reverse transcriptase inhibitors Nucleoside reverse transcriptase inhibitors (NRTIs) Protease inhibitors Fusion inhibitors CCR5 coreceptor antagonists Integrase inhibitors

NS

Normal Saline

Fungal infections have increased for what reason?

Number of immunocompromised patients - AIDS and ARC - taking transplantation surgery - cancer treatment

A 3-month-old infant has been diagnosed with respiratory syncytial virus (RSV) and will begin treatment with inhaled ribavirin. Which of the following measures should be taken in the care of this patient and administration of ribavirin?

Nurses who are pregnant or who may be pregnant should not administer the drug.

Which drug is used to treat thrush/ "swish and swallow"

Nystatin

A client with an infected ulcer on the foot has been prescribed daptomycin. What action should the nurse ensure has been taken to determine that the drug will be effective in treating the infection?

Obtain a culture of the client's infection.

Your patient has been prescribed Clotriamazole to treat a tinea infection. Why should you avoid occulsive bandages in this patient?

Occulsive bandages could cause the drug to be systemically absored and these drugs are too toxic to be absorbed into the body.

What is something important to assess in a patient before giving them Chloroquine?

Opthalamic/ visual exam

A nurse is preparing a presentation for a local community group after media reports about cases of avian influenza in the surrounding region. When explaining the treatment options for avian flu, the nurse should reference what drug?

Oseltamivir

Many of the residents of a long-term care facility have become sick with influenza this winter despite the fact that all residents were immunized earlier in the fall. A nurse who provides care in the facility should recognize the need to have which of the following drugs readily available at the facility?

Oseltamivir phosphate

Adverse effects

Ototoxicity and Nephrotoxicity are most significant

Main aminoglycoside adverse effects are

Ototoxicity and nephrotoxicity

Example: Label the IV Order: 500 mL D5W 1 / 2 NS Rate: 50 mL/hr.

PATIENT NAME: BRIYANA JACKSON ROOM: 1234B BED NUMBER: 2 DATE: 8/26/15 TIME: 1306 (1:06PM)

Michael, 25 years old, has had mitral valve regurgitation since age four, after having rheumatic fever. Michael is planning to go to his dentist to have his teeth cleaned. Because of Michael's history he will need to take antibiotics in conjunction with this procedure to prevent bacteremia. Which class of antibiotics will Michael most likely receive if he has no allergies?

Penicillin

What is the most common opportunistic respiratory infection in patients with AIDS?

Pneumocystis Jiroveci Pneumonia

If child needs to travel, the CDC or local health department should be contacted about what

Preventative measures

When providing care to a client with a viral infection, the nurse knows that ribavirin (Virazole) for inhalation is used to treat which virus?

RSV

The nurse is teaching a client about his prescription for a macrolide antibiotic. What should the nurse teach the client to report?

Recurring symptoms of infection

A client returns from a vacation with traveler's diarrhea. The nurse understands that what is the treatment of choice?

Rifaximin

Six Rights of Safe Drug Administration

Right patient right drug right dose right route right time right documentation

Prior to and following a transurethral prostatic resection (TUPR), a 73-year-old man has been scheduled to receive a total of five intravenous doses of cefazolin. Which of the following nursing diagnoses should the nurse associate with this antibiotic therapy?

Risk for Infection related to overgrowth of nonsusceptible organisms

Medications for for Intermittent IV administrations

Some Iv medications are given not continuously but intermittently such as every 4,6,8 hours. this is known as IVPB. To solve IVPB problems, you can use the calculation much like the one needed for the IV: mL x TF ------------ = gtt/min minutes mL= given TF= normally macro, but given min= this may or may not be given, when it is not given however follow this general rule for adults; allow 30 mintues for ever 50 mL of solution.

Injections From Powders

Some medications are prepared in a dry form, powder or crystal. As for Liquids they are unstable and they lose potency overtime . The drug must be reconstituted according to the manufacturer's directions which will give the type and the amount of diluent to use. To solve injection from powders problems , you use the same rule as for oral medication and injection from a liquid. *you use formula method. this is easiet for you *

When Supply is a Ratio

Sometimes the Labels can state the supply as a ratio Ratios are always interpreted in the metric system as GRAMS PER mL Example: 1:1000 means 1 g in 1000 mL It can be stated in three different ways : 1 g per 1000 mL 1 g = 1000 mL 1 g/1000 mL

What do protein kinase inhibitors act on?

Specific enzymes that are needed for protein building by specific tumor cells

A client is prescribed ciprofloxacin for a urinary tract infection. The nurse is preparing to teach the client about the medication. What must the nurse include in the education plan?

Take precautions to prevent photosensitivity.

A physician orders 100 units of regular insulin in 100 mL to infuse at 10 mL per hour. How many units will the patient receive each hour?

The IV is 100 units in 100 mL or 1 Unit/mL. They receive 10 mL per hour so he or she receives 10 units each hour 100 ------ = 1 unit/ mL x 10 = 10 units 100

An immigrant and his wife are both HIV positive. The physician prescribes antiretroviral medications. However, the client, tell the nurse he is concerned about his ability to afford the treatment. What advice can the nurse provide to the client to help begin the treatment?

The client can approach social agencies and pharmaceutical companies that provide antiretroviral drugs on compassionate grounds

The nurse is caring for a patient who has HIV and is taking saquinavir therapy. What should the nurse include in the dietary education for this patient?

The patient should not take this medication with grapefruit or pomegranate juice.

How do alkylating agents produce their cytoxic effects?

The react chemically with portions of the RNA, DNA, or other cellular proteins, being most potent when they bind with cellular DNA

1935:

The sulfonamides were introduced

A client is administered a nucleotide reverse transcriptase inhibitor in combination with a nonnucleotide reverse transcriptase inhibitor. What is the rationale when administering these medications together?

They have synergistic antiviral effects.

How are Antineoplastic antibodies cytoxic?

They interfere with cellular DNA synthesis by inserting themselves between base pairs in the DNA chain

How do mitotic inhibitors interfere?

They interfere with the ability of a cell to divided by blocking DNA synthesis, which causes cell death

What is the intended action of antineoplastic drugs?

They target abnormal cells that compose the neoplasm or cancer, which has a greater impact on them than normal cells. Antineoplalstic drugs alter human cells in a variety of ways

________ should be dissolved slowly into the mouth

Troches

Where are protozoal infections most common?

Tropical areas of the world. It is common to see many people suffer multiple infestations at the same time

A client may experience an acute exacerbation of hepatitis B if entecavir, adefovir, or telbifudine therapy is stopped.

True

True or False: Cancer is considered the second leading cause of death in the United States.

True (Rationale: Cancer is the second leading cause of death. Coronary disease is the leading cause of death.)

True or False: The mainstay for treatment for malaria is chloroquine.

True (Rationale: Chloroquine is currently the mainstay of antimalarial therapy, but many strains of the parasite are developing resistance to the drug, so the CDC often recommends the use of certain antibiotics as part of combination therapy.)

True or False: A patient with alopecia should cover his or her head if he or she will be exposed to extremes of temperature.

True (Rationale: Some patients may choose to get a wig as soon as their hair begins to fall out while others may prefer to remain bald. Whichever choice the patient makes, it is important to teach the patient to cover his or her head in extremes of temperature or when out in the sun for extended periods of time, because a great deal of heat is lost through the head in cold weather and the skin is very subject to sunburn in warm, sunny weather.)

True or False: An adverse effect of tamoxifen therapy involves menopausal symptoms.

True (Rationale: Tamoxifen is a hormone modulator and is antiestrogen, blocking estradiol production without effects on adrenal hormones. As a result, menopausal symptoms like hot flashes, mood swings, edema, and vaginal dryness and itching can occur.)

True or False: A topical antifungal cream should be gently rubbed into the affected area that has been washed with soap and water and patted dry.

True (Rationale: The area should be gently cleaned before applying the antifungal cream and then gently rubbed into the affected area. Avoid covering the area to reduce risk of systemic absorption. Patients should be taught to wash their hands after application [unless the cream was applied to the hands].)

True or False: Crowded unsanitary conditions help contribute to protozoal infections.

True (Rationale: Unsanitary conditions contribute to protozoal infections, especially giardiasis and amebiasis.)

Sarcoma

Tumor that originates in the mesenchyme and is made up of embryonic connective tissue

Carcinoma

Tumor; originates in epithelial cells

Patients receiving anti protozoal infections should REALLY be monitored for what?

Vision loss/retinal damage Ototoxicity

The nurse is preparing a teaching plan for a client who is receiving cephalosporins. Which of the following would the nurse identify as the most commonly occurring adverse effects?

Vomiting and diarrhea

W

Water

What should you advise a patient to do who is taking Terbinafine to treat his athletes foot?

Wear clean dry socks

Adding Medications to IVs

When a continuous IV order includes a medication, generally this medication arrives already premixed in the infusion bag or the pharmacist can add it on site. Sometimes as the nurse, we have to add the medication ourselves and determine the rate of flow. if this happens: first calculate how much of the medication to add to the IV fluids then calculate the drip rate.

When could your patient stop taking Metronidazole?

When his stool sample comes back negative

Clearing Decimals When using the Formula Method:

When the numerator and the denominator are decimals, add zeros to make the number of decimal places the same. Then drop the decimal points. This short operation replaces long division. example: 0.5 mg / 0.25 mg add the zero 0.50 mg / 0. 25 mg.

Microdrip

When the set is small, and the drops are small as well all sets deliver 60 drops per mL (60 gttt/mL)

Syringes and Rounding

When you are calculating injection answers, the degree of accuracy depends on the syringe that you use. REMEMBER: The 3 mL syringe answers are in the nearest hundredth, but you round to the nearest tenth. example: 1.25 becomes 1.3 mL The 1 mL syringe the answers are in the nearest thousandths, but you round to the nearest hundredth. example: 0.978 becomes 0.98 mL.

FACT!!

When your using an infusion pump for IVPB, solve the problem by setting the TF to 60 gtt

A female HIV-positive patient with a high CD4 count has been taking nevirapin (Viramune) for 9 weeks. What clinical manifestations indicate to the nurse that the patient is having complications from this medication?

Yellow sclera and hepatomegaly

A pregnant client's prenatal blood work has returned positive for HIV. What antiretroviral medication would be safest to give this client?

Zidovudine

A patient is prescribed terbinafine for treatment of onychomycosis of the toenail. The nurse would reinforce the need for the patient to continue this treatment for what length of time? a. 12 weeks b. 6 weeks c. 8 weeks d. 10 weeks

a. 12 weeks (Rationale: For onychomycosis of the toenail, terbinafine is used for 12 weeks. Treatment lasting less than 12 weeks is less likely to be effective.)

When describing mycosis to a group of students, what would the nursing instructor include? a. A disease that is caused by a fungus b. A superficial infection of the skin related to fungal invasion c. An underlying immune disorder responsible for the symptoms d. An infection involving an overgrowth of opportunistic organisms

a. A disease that is caused by a fungus (Rationale: Mycosis refers to a disease that is caused by a fungus. Mycosis is unrelated to immune disorders. A superinfection refers to an infection that involves an overgrowth of opportunistic organisms. In some cases, a superinfection can be due to a fungal infection. Mycosis refers to a fungal infection that can be localized, superficial, or systemic.)

The nurse sees a patient in the clinic who has been taking chloroquine for the treatment of malaria. While the nurse measures vital signs, the patient repeatedly rubs her eyes. When the nurse questions why, the patient says, "I guess it's time for a trip to the eye doctor. My glasses don't seem to work very well and I'm having trouble with my vision." What does the nurse suspect is happening with this patient? a. Adverse effect of medication b. Muscles controlling the eye are impacted by malaria c. Chemical actions of the medication are reducing aqueous humor d. Malaria infection is damaging optic tissue

a. Adverse effect of medication (Rationale: Adverse effects of chloroquine include visual disturbances and retinal changes, so this patient should be encouraged to see an ophthalmologist immediately to prevent further damage. Malaria does not impact the optic tissue nor is it likely to impact the muscles controlling eye movement. The drug does not impact the production of aqueous humor.)

The nurse recognizes that nitrogen mustards belong to what classification of antineoplastic? a. Alkylating agent b. Hormone modulator c. Mitotic inhibitor d. Antimetabolite

a. Alkylating agent (Rationale: Nitrogen mustard is among the oldest drugs in the classification of alkylating agents, and modification of the structure of this and other older drugs has led to the development of nitrosoureas. Therefore, the other options are incorrect.)

The nurse would classify what types of cancer as solid tumors? (Select all that apply.) a. Carcinomas b. Lymphoma c. Hematologic malignancies d. Sarcomas e. Leukemia

a. Carcinomas d. Sarcomas (Rationale: Cancers can be divided into two groups: solid tumors and hematological malignancies. Solid tumors may originate in any body organ and may be further divided into carcinomas, or tumors that originate in epithelial cells, and sarcomas, or tumors that originate in the mesenchyme and are made up of embryonic connective tissue cells. Hematological malignancies, such as the leukemias and lymphomas, occur in the blood-forming organs.)

A nurse is presenting a seminar on protozoal infections to a community group that travels to various parts of the world. They are currently planning a trip to South America. As part of the seminar the nurse discusses Chagas disease, teaching the group that this infection is transmitted by what? a. Common house fly b. Sand fly c. Drinking of unpurified spring water d. Tsetse fly

a. Common house fly (Rationale: Chagas disease, almost endemic in many South American countries, is transmitted by the common house fly. African sleeping sickness is transmitted by the tsetse fly. Leishmaniasis is transmitted by the sand fly. Giardiasis is associated with the drinking of unpurified spring water.)

How would the nurse describe the action of alkylating agents? a. React chemically with portions of RNA, DNA, and other cellular proteins b. Block DNA synthesis to interfere with the cell's ability to divide c. Inhibit DNA production by replacing the natural substances for cell function d. Insert itself between base pairs in the DNA chain to disrupt DNA synthesis

a. React chemically with portions of RNA, DNA, and other cellular proteins (Rationale: Alkylating agents react chemically with portions of the RNA, DNA, or other cellular proteins. Antimetabolites inhibit DNA production in cells that depend on certain natural metabolites to produce DNA, replacing the needed metabolites, which prevents normal cellular function. Mitotic inhibitors interfere with the ability of a cell to divide, blocking or altering DNA synthesis. Antineoplastic antibiotics interfere with cellular DNA synthesis by inserting themselves between base pairs in the DNA chain.)

A patient is prescribed mefloquine as part of antimalarial treatment. Before administering this medication, the nurse would need to know the action of this drug is what? a. Rupture of the cell leading to its death b. Blockage of folic acid used by the parasite c. Disruption of the parasite's mitochondria d. Interference with the parasite's reproductive ability

a. Rupture of the cell leading to its death (Rationale: Mefloquine increases the acidity of the plasmodial food vacuoles, causing cell rupture and death. Primaquine acts to disrupt the mitochondria. Chloroquine changes the metabolic pathways needed for parasite reproduction. Pyrimethamine blocks the use of folic acid needed for protein synthesis by the parasite.)

The patient is diagnosed with a candida infection of the mouth, which the nurse documents as what? a. Thrush b. Ringworm c. Dermatophytes d. Tinea cruris

a. Thrush (Rationale: Thrush is a candida infection of the mouth, often identified by small white spots on the tongue most commonly seen in newborns or patients who are immunocompromised. Ringworm is caused by tinea infection, not candida, and is usually found on the skin and not in the mouth. Fungi known as dermatophytes include tinea infections, of which candida infections are only one small subgrouping. Tinea cruris is also called jock itch and occurs in the area of the genitalia, usually as the result of perspiration from athletic activities.)

NRTI

abacavir emtricitabine lamivudine didanosine stavudine tenofovir zidovudine

intramuscular

absorbed more rapidly than subcutaneous or intradermal, angle of injection 90 degrees ASPIRATE

NRTI actions and indications

actions: compete with naturally occurring nucleosides within cell that virus would use to build DNA chain indications: management of symptomatic HIV

integrase inhibitors actions & indications

actions: inhibits activity of integrase. blocking this prevents formation of HIV1 and leads to decrease in viral load and increase in active CD4 cells indications: in combo with other meds for treatment of HIV

anti hep b agents actions & indications

actions: inhibits hep b virus reverse transcriptase indications: hep b

agents for herpes & cyto actions and indications

actions: inhibits viral DNA replication indications: herpes

fusion inhibitors actions and indications

actions: prevents fusion of virus with the human cellular membrane indications: HIV 1 pt who have experienced clinical or immunological deterioration after treatment with other agents

agents for influenza A indications & actions

actions: prevents shedding of viral protein coat indications: influenza A

agents for herpes & cytomegalovirus

acyclovir cidofovir famciclovir foscarnet ganciclovir valacyclovir valganiclovir

Order: 250 mg in 250 mL ; run at 50 mL/hour for 1 hour Available: ampule labeled 1 g in 10 mL; Microdrip 60 gttt/min

add 2.5 mL to the Iv 250 mg / 1000 mg x 10 mL = 2.5 Micro: 50 x 60 ----------- = 50 60 min

agents for influenza A contraindications

allergy renal impairment pregnancy

The nurse is to administer enfuvirtide (Fuzeon) to a 27-year-old man. After the medication has been reconstituted with sterile water, the nurse will:

allow the solution to sit until it dissolves

agents for influenza A & resp virus

amantadine oseltamivir peramivir ribavirin rimantadine zanamivir

What antiviral drug treatment should the nurse be prepared to administer to manage an outbreak of influenza A among residents of a long-term care facility?

amantadine hydrochloride

when to use 3 mL syringe

amounts 1-3 mL

when to use 1 mL syringe

amounts less than 1 mL

A 10-year-old client is diagnosed with acute otitis media. What does the nurse anticipate the health care provider will prescribe?

amoxicillin

The nurse should advise the client to avoid taking which medication at the same time as a fluoroquinolone?

antacids

A female patient is receiving mefloquine as part of malarial prevention. After teaching about the drug, which statement by the patient indicates successful teaching? a. "I should take the drug once a day for 4 weeks." b. "I should avoid getting pregnant for at least 2 months after I finish the drug." c. "I need to take the drug on an empty stomach even if the drug upsets my stomach." d. "I need to call the clinic if I develop a rash or headache."

b. "I should avoid getting pregnant for at least 2 months after I finish the drug." (Rationale: Mefloquine is teratogenic in preclinical studies; pregnancy should be avoided during and for 2 months after completion of therapy. Typically prevention therapy would require the patient to take the drug once a week starting 1 week before travel and continuing for 4 weeks after leaving the area. The drug can be taken with food or meals if GI upset occurs. Rash and headache are common adverse effects of the drug and do not need to be reported.)

A nurse administers griseofulvin at 9 a.m. At which time would the nurse expect the drug to peak? a. 12 a.m. b. 1 p.m. c. 10 a.m. d. 11 a.m.

b. 1 p.m. (Rationale: Griseofulvin peaks in 4 hours, making the peak time around 1 p.m.)

A patient who is planning to travel to Africa is prescribed chloroquine to prevent malaria. The nurse would instruct the patient to begin the therapy at approximately which time? a. The day after returning from the trip b. 2 weeks before possible exposure c. Upon arrival at the destination d. 1 to 3 days before leaving on the trip

b. 2 weeks before possible exposure (Rationale: Chloroquine is usually started 1 to 2 weeks before exposure and then continues for 4 weeks after leaving the area.)

When a cell loses its differentiation and organization, the nurse recognizes that what has occurred? a. Autonomy b. Anaplasia c. Neoplasm d. Metastasis

b. Anaplasia (Rationale: Anaplasia is the loss of organization and structure. Anaplasia is a property held by all cancer cells. Autonomy is the loss of the normal controls and reactions that inhibit growth and spreading, which is another property of cancer cells. Metastasis is the ability of a cancer cell to enter the circulatory or lymphatic system and travel to other areas of the body that are conductive to growth and survival, which is another property of some cancer cells. A neoplasm is a new or cancerous growth that occurs when abnormal cells have the opportunity to multiply and grow.)

The creation of blood vessels to supply oxygen and nutrients to the abnormal cells is known as what? a. Anaplasia b. Angiogenesis c. Carcinoma d. Autonomy

b. Angiogenesis (Rationale: Angiogenesis is the creation of new blood vessels that occurs when cancer cells release an enzyme that promotes new vessels to feed the cancer cells. Anaplasia is the loss of organization and structure. Anaplasia is a property held by all cancer cells. Autonomy is the loss of the normal controls and reactions that inhibit growth and spreading, which is another property of cancer cells. Carcinoma is a term used to define a tumor that originates from epithelial cells.)

What is the nurse's priority teaching point for a patient who is receiving temsirolimus? a. Drink adequate fluids b. Avoid grapefruit juice c. Promote frequent rest periods d. Maintain adequate nutrition

b. Avoid grapefruit juice (Rationale: Although rest, fluids, and nutrition are important for any patient receiving an antineoplastic, a patient receiving temsirolimus needs to avoid grapefruit juice because of possible drug-food interaction, and this is the priority teaching point.)

After reviewing the various antifungal agents, the students demonstrate understanding of the material when they identify which agent as a topical agent? a. Itraconazole b. Butoconazole c. Voriconazole d. Fluconazole

b. Butoconazole (Rationale: Butoconazole is a topical antifungal agent. Fluconazole, itraconazole, and voriconazole are systemic antifungal agents.)

A patient is receiving idarubicin. What is the nurse's priority assessment? a. Blood studies b. Cardiac function c. Electrolyte levels d. Respiratory function

b. Cardiac function (Rationale: The patient's cardiac function needs to be monitored closely because idarubicin is specifically toxic to the heart. The response of "blood studies" is nonspecific and could involve virtually any lab test. Respiratory function is not commonly impacted by idarubicin. Electrolyte levels should be monitored in patients receiving any antineoplastic study due to adverse effects impacting nutrition and kidney function, but are not specific to idarubicin.)

The nurse suspects a patient with Plasmodium falciparum malaria will have difficult complying with pharmacotherapy if he has to take several different pills per day. What approach might improve compliance? a. Pediatric dosing b. Combination therapy c. Hospitalizing the patient d. Prophylactic therapy

b. Combination therapy (Rationale: Combination therapy can be helpful in improving compliance because there are fewer pills to take and the medications are taken less frequently. Prophylactic therapy would not be helpful because both prophylactic and pediatric doses tend to be smaller and would not treat an existing condition. Hospitalizing a patient just to give medications is not cost effective and would not be an option.)

The patient taking chloroquine reports blurred vision to the clinic nurse. What nursing diagnoses would be appropriate for this patient? (Select all that apply.) a. Risk for infection related to visual damage b. Disturbed sensory perception related to blurred vision c. Risk for injury related to blurred vision d. Deficient knowledge regarding drug therapy e. Acute pain related to visual disturbance

b. Disturbed sensory perception related to blurred vision c. Risk for injury related to blurred vision (Rationale: The patient has disturbed sensory perception in the form of blurred vision and this puts the patient at risk for injury. There is no indication of deficient knowledge regarding drug therapy, acute pain, or risk for infection.)

After teaching a group of students about antineoplastic agents and their effect on cancer and healthy cells, the instructor determines that the teaching was successful when the students identify which drug as having the least effect on healthy cells? a. Vinblastine b. Gefitinib c. Docetaxel d. Paclitaxel

b. Gefitinib (Rationale: Gefitinib belongs to the group of drugs called protein tyrosine kinase inhibitors, which do not affect healthy human cells. Docetaxel is a mitotic inhibitor that affects both healthy and cancer cells. Vinblastine is a mitotic inhibitor that affects both healthy and cancer cells. Paclitaxel is a mitotic inhibitor that affects both healthy and cancer cells.)

What would the nurse expect the health care provider to prescribe for treatment of tinea infection? a. Flucytosine b. Griseofulvin c. Amphotericin B d. Nystatin

b. Griseofulvin (Rationale: Griseofulvin is indicated for the treatment of a variety of tinea infection caused by susceptible Trichophyton species. Amphotericin B is indicated for the treatment of aspergillosis, leishmaniasis, cryptococcosis, blastomycosis, moniliasis, coccidioidomycosis, histoplasmosis, and mucormycosis. Flucytosine is indicated for the treatment of systemic infections caused by Candida or Cryptococcus. Nystatin is indicated for treatment of candidiasis [oral form]; treatment of local candidiasis, vaginal candidiasis, and cutaneous and mucocutaneous infections caused by Candida species.)

While receiving etoposide IV, extravasation occurs. What agent would the nurse expect to have ordered to reduce tissue damage? a. Sodium bicarbonate b. Hyaluronidase c. Isotonic sodium thiosulfate d. Ascorbic acid

b. Hyaluronidase (Rationale: Hyaluronidase is used to extravasations of etoposide, teniposide, vinblastine, or vincristine. Sodium bicarbonate is used for extravasations of daunorubicin, doxorubicin, vinblastine, or vincristine. Ascorbic acid is used for extravasations of dactinomycin. Isotonic sodium thiosulfate is used for extravasation of mechlorethamine.)

A patient with recurrent vaginal yeast infection is to receive tioconazole. The nurse would instruct the patient to do what? a. Apply the cream to the area twice daily. b. Insert a full applicator at bedtime only once. c. Use the powder as needed. d. Insert the suppository for at least 3 consecutive days.

b. Insert a full applicator at bedtime only once. (Rationale: Tioconazole is supplied as a vaginal ointment meant for one dose treatment only. A full applicator of the ointment is inserted vaginally at bedtime.)

A patient with trypanosomiasis is to receive pentamidine. The nurse would expect the health care provider to order this drug to be administered by which route? a. Inhalation b. Intravenous c. Oral d. Topical

b. Intravenous (Rationale: For the treatment of trypanosomiasis, pentamidine is given IV or IM. Inhalation form is used for treating pneumocystis carinii pneumonia. Pentamidine is not administered orally. Pentamidine is not administered topically.)

What medication would the nurse classify as something other than an echinocandin antifungal agent? a. Anidulafungin b. Terbinafine c. Micafungin d. Caspofungin acetate

b. Terbinafine (Rationale: Terbinafine is an example of an azole antifungal agent. Anidulafungin, caspofungin, and micafungin are examples of echinocandin antifungal agents.)

What antibiotics might the nurse administer in combination with antimalarial medications to treat a resistant strain of plasmodium? (Select all that apply.) a. Erythromycin b. Tetracycline c. Clindamycin d. Cefazolin e. Doxycycline

b. Tetracycline c. Clindamycin e. Doxycycline (Rationale: Doxycycline, tetracycline, and clindamycin may be administered with antimalarials like quinine in order to treat resistant strains of plasmodium. Erythromycin and cefazolin have not been found to be effective.)

Insulin is usually administered:

by subcutaneous injection. Often check blood glucose 30 minuets before meals and at bed time.

A patient asks the nurse how long he should use the prescribed naftifine. Which response would be the most appropriate? a. "No longer than 6 weeks." b. "No longer that 8 weeks." c. "No longer than 4 weeks." d. "No longer than 2 weeks."

c. "No longer than 4 weeks." (Rationale: Naftifine should be used for no longer than 4 weeks due to the risk of adverse effects and possible emergence of resistant strains of fungi.)

A patient is receiving gentian violet as topical treatment for a fungal infection. What instruction would be most important for the nurse to include in the teaching plan? a. "If you get any of the medicine on your clothes, wash them in cold water." b. "Cover the area with plastic wrap to seal it off." c. "This medication can stain your skin bright purple." d. "Apply this medicine to any open active lesions."

c. "This medication can stain your skin bright purple." (Rationale: Gentian violet stains the skin and clothing bright purple. It is difficult to remove no matter what temperature water you wash clothes in. Occlusive dressing should be avoided to prevent systemic absorption; gentian violet is very toxic when absorbed. Gentian violet should not be applied to open lesions.)

What classification of antineoplastic agents would be considered cancer non-cell cycle specific agents? a. Antimetabolites b. Protein tyrosine kinase inhibitors c. Alkylating agents d. Mitotic inhibitors

c. Alkylating agents (Rationale: Alkylating agents affect cells in all phases of the cell cycle and are considered non-cell cycle specific. Antimetabolites are considered to be S phase specific agents. Mitotic inhibitors are cell cycle-specific agents working in the M phase of the cell cycle. Protein tyrosine kinase inhibitors target specific enzymes needed for protein building by specific tumor cells. They do not affect health human cells.)

A patient is experiencing antineoplastic-induced nausea and vomiting. The health care provider prescribes ondansetron. The nurse understands that this agent acts in what way? a. Blocking human substance P/neurokinin 1 receptors b. Calming the activity of the gastrointestinal tract c. Blocking serotonin receptors in the CTZ d. Directly blocking the chemoreceptor trigger zone (CTZ)

c. Blocking serotonin receptors in the CTZ (Rationale: Ondansetron blocks serotonin receptors in the CTZ. Aprepitant blocks human substance P/neurokinin 1 receptors in the central nervous system. Benzodiazepines directly block the CTZ. Metoclopramide calms the activity of the GI tract.)

What would a nurse incorporate into the teaching plan for a patient who is receiving fluconazole about how the drug works? a. Inhibits the cytochrome P2D6 enzyme system b. Inhibits the synthesis of ergosterol to prevent cell wall formation c. Blocks the activity of a sterol in the fungal wall d. Inhibit the synthesis of glucans

c. Blocks the activity of a sterol in the fungal wall (Rationale: Fluconazole blocks the activity of a sterol in the fungal wall. Terbinafine inhibits the CYP2D6 enzyme system. Posaconazole and voriconazole inhibit the synthesis of ergosterol, leading to an inability of the fungus to form a cell wall. The echinocandins work by inhibiting glucan synthesis.)

When describing alkylating agents, the nurse identifies what drug as a prototype? a. Fluorouracil b. Vincristine c. Chlorambucil d. Methotrexate

c. Chlorambucil (Rationale: Chlorambucil is the prototype alkylating agent. Fluorouracil is an antimetabolite. Methotrexate is the prototype antimetabolite. Vincristine is the prototype mitotic inhibitor.)

The nurse is teaching peers about fungal infections and describes the cell membrane of a fungal cell how? a. Has a soft cell wall b. Contains monosaccharides c. Contains ergosterol d. Contains mycosis

c. Contains ergosterol (Rationale: Ergosterol is a steroid-type protein found in the cell membrane of fungi; similar in configuration to adrenal hormones and testosterone. Fungi have a hard cell wall. The cell membrane contains polysaccharides. Mycosis is a disease caused by a fungus.)

A patient who is receiving drug therapy for treatment of giardiasis asks the nurse about how she may have gotten this infection. The nurse would incorporate knowledge of transmission of the infection by the following when responding to the patient? a. Sexual intercourse b. Common house fly c. Contaminated water d. Mosquito bite

c. Contaminated water (Rationale: Giardiasis is transmitted by the ingestion of contaminated water or food. Trichomoniasis typically is transmitted by sexual intercourse with men who have no signs and symptoms of the infection. Malaria is transmitted by a mosquito bite. Chagas disease is transmitted by the common house fly.)

The patient is receiving ketoconazole. The nurse would be concerned about a drug-drug interaction if the patient was prescribed what other medication? a. Dofetilide b. Lovastatin c. Digoxin d. Pimozide

c. Digoxin (Rationale: Ketoconazole interacts with digoxin, resulting in an increased serum level of digoxin. The other drugs interact with itraconazole.)

A patient is receiving a toremifene. What signs and symptoms will the nurse be especially alert for that are specific to this drug? a. CNS changes b. Infection c. Hypercalcemia d. Renal dysfunction

c. Hypercalcemia (Rationale: Toremifene is a hormone/hormone modulator that often produces hypercalcemia. Many antineoplastic drugs produce bone marrow suppression placing the patient at high risk for infection, so that is not specific to this drug. Many antineoplastics have CNS changes as a common side effect. Renal dysfunction is important to dosage management of almost all antineoplastic drugs and is not specific to toremifene.)

A nurse is administering an antineoplastic that can cause necrosis and cellulitis if extravasation occurs. Which intervention would be most appropriate to prevent this from occurring? a. Using an infusion pump b. Administering through a hand vein c. Inspecting site frequently for redness or swelling d. Using proximal veins

c. Inspecting site frequently for redness or swelling (Rationale: Site inspection is a major intervention for preventing extravasation. Distal veins should be used. Small veins in the hand or wrist should be avoided. An infusion pump should be avoided because it can continue to administer under pressure, leading to severe extravasation.)

A nurse is reviewing the medical record of a patient for whom antimalarial therapy is being proposed. The nurse would consider a history of what as a contraindication for use? a. Psoriasis b. Retinal disease c. Liver disease d. Porphyria

c. Liver disease (Rationale: A history of liver disease is a contraindication because of the parasitic invasion of the liver causing damage and the metabolism of the drug, which could further contribute to hepatotoxicity. A history of porphyria or psoriasis would require cautious use and close monitoring for possible skin damage. A history of retinal disease would require cautious use and close monitoring of vision and retina.)

What agents would be considered to have the best gametocytocidal activity? a. Pyrimethamine b. Mefloquine c. Primaquine d. Chloroquine

c. Primaquine (Rationale: Primaquine is the most gametocytocidal agent. Chloroquine and mefloquine are schizontocidal. Pyrimethamine is a sporontocidal agent.)

A patient who develops trypanosomiasis is to receive pentamidine. Based on the nurse's knowledge of the infection and drug therapy, what is the priority nursing diagnosis likely to be for this patient? a. Acute pain b. Diarrhea c. Risk for injury d. Imbalanced nutrition less than body requirements

c. Risk for injury (Rationale: Although all the nursing diagnoses may be appropriate, the CNS effects associated with the infection coupled with the CNS adverse effects of the drug would make the nursing diagnosis of risk of injury the priority.)

The nurse is preparing to administer ondansetron to treat a patient experiencing nausea and vomiting as a result of antineoplastic therapy. The nurse looks up the medication and learns that this agent will block what chemical? a. Norepinephrine b. Dopamine c. Serotonin d. GABA

c. Serotonin (Rationale: Ondansetron blocks serotonin receptors in the chemoreceptor trigger zone. It has no effect on blocking dopamine, norepinephrine, or GABA.)

When describing the various effects of antineoplastic agents, the nurse explains that antineoplastic drugs primarily affect human cells that are rapidly multiplying, going through the cell cycle quickly. The nurse would identify what cells as an example? a. Breast b. Ovaries c. Skin d. Testicles

c. Skin (Rationale: Skin cells proceed very rapidly through the cell cycle. Breast cells, cells of the testicles, and cells of the ovaries proceed very slowly through the cell cycle.)

The nurse is caring for a patient taking antimalarials for prophylaxis while serving in the Peace Corps in Africa. The patient has taken the medication for 2 months and is continuing to lose significant weight due to the GI effects of the drug. What recommendations can the nurse make to reduce GI adverse effects and promote healthy nutrition for this patient? (Select all that apply.) a. Avoid alcohol. b. Add extra fat to diet for calories. c. Take the drug immediately after meals. d. Eat 5 to 6 small meals a day. e. Lie down immediately after eating.

c. Take the drug immediately after meals. d. Eat 5 to 6 small meals a day. (Rationale: The patient should be taught to take the drug with food and eat five or six small meals a day instead of three large meals to avoid overdistention of the stomach. Alcohol should be avoided due to possible CNS effects but that is not related to GI adverse effects. Lying down after eating would increase risk of gastroesophageal reflux and would not reduce GI adverse effects. Fatty foods tend to increase risk of nausea so this would not be appropriate for the patient.)

What medication can the nurse administer that is available in a sprinkle formulation for children? a. Nystatin b. Flucytosine c. Terbinafine d. Griseofulvin

c. Terbinafine (Rationale: Terbinafine is available in a sprinkle formation useful with children who cannot swallow pills. Nystatin is available in liquid formula for treating oral thrush in newborns but is not available in sprinkle formulation. Griseofulvin is safe to use in children but it is not available in sprinkle formulation. Flucytosine is available in pill form.)

insulin syringe

calibrated in units: 30, 50, 100. Always only use insulin syringe for insulin.

5 mL and 10 mL syringe

calibrated to 0.2 mL increments

1 mL syringe

calibrated to nearest hundredths place

3 mL syringe

calibrated to tenths

Fungi ________ cause infection

can cause infection!!!

Fluroquinolone drugs include

ciprofloxacin (Cipro)

maraviroc

class: CCR5 co antagonist indications: combo antiretroviral treatment of adults infected with CCR5 tropic HIV1 who have evidence of viral replication and HIV 1 strains resistant to multiple antiretroviral agents actions: binds to CCR5 on cell membrane preventing interaction of HIV1 & CCR5 adverse effects: dizzy, paresthesias, nausea, vomiting, diarrhea, cough, UTI, fever, musculoskeletal symptoms, hepatoxicity

zidovudine (retrovir)

class: NRTI indications: HIV in combo with other antiretrovirals prevention of maternal - fetal HIV transmission actions: thymidine analogue that is activated to a triphosphate form inhibiting replication of various retroviruses adverse effects: headache, insomnia, dizzy, nausea, diarrhea, fever, rash, bone marrow suppression

acyclovir (sitavig)

class: agents for herpes indications: herpes simplex virus 1&2 actions: inhibits viral DNA replication adverse effects: headache, vertigo, tremors, nausea, vomiting, rash

adefovir (hepsera)

class: anti hep b agents indications: chronic hep b in adults with evidence of viral replication and evidence of persistent elevations in alanine aminotransferase and asparate aminotransferase/histologically active disease actions: inhibits hep b virus reverse transcriptase, causes DNA chain termination, blocks viral replication adverse effects: headache, asthenia, nausea, fatal hepatomegaly with steatosis, nephrotoxicity, lactic acidosis, exacerbation of hep b when discontinued

enfuvirtide

class: fusion inhibitor, agents for HIV indications: HIV 1 pt who have experienced clinical or immunological deterioration after treatment with other agents actions: prevents entry of HIV 1 virus into cells by inhibiting fusion of the virus membrane with cellular membrane adverse effects: headache, nausea, vom, diarrhea, rash, anorexia, pneumonia, chills, injection site reaction

raltegravir

class: integrase inhibitor agents for HIV indications: in combo with other antiviral agents for treatment of HIV 1 in pt who have viral replication and HIV 1 strains resistant to multiple antiretroviral agents actions: inhibits activity of integrase. blocking this prevents formation of HIV 1 and leads to decrease in viral load and increase in active CD4 cells adverse effects: headache, dizzy, nausea, vom, diarrhea, fever, rhabdomyolysis

nevirapine (viramune)

class: nonNRTI/agents for HIV indications: HIV treatment actions: binds to HIV 1 and blocks replication adverse effects: headache, nausea, vomiting, diarrhea, rash, liver dysfunction, chills, fever

fosamprenavir (lexiva)

class: protease inhibitors agents for HIV indications: HIV adverse effects: headache, rash, stevens johnson syndrome, redistribution of body fat action: inhibits protease activity leading to formation of immature, noninfectious virus particles

Antimalarial drugs are given in _______________ form to attack the Plasmodium

combination

Giardiasis is caused by what?

contaminated water

Cancer remains second to which disease as the leading cause of death in US?

coronary disease

medications that are given through an enternal tube must be _______

crushed or in liquid form. Always flush with 30mL after

A client with a gram-negative bacterial infection has been prescribed a fluoroquinolone. During the preadministration assessment, the nurse should ensure that which laboratory tests, if prescribed, are obtained before the first dose of fluoroquinolones is administered to the client?

culture and sensitivity test

After teaching a group of students about antimalarial therapy, the instructor determines that the teaching was successful when the students make what statement? a. "Quinine is still considered to be the most important drug used for treatment." b. "Drug therapy for malaria has virtually extinguished any resistant strains." c. "Usually, one drug is adequate to destroy the protozoa in the early stages." d. "Combination therapy attack the parasite at different stages of its life cycle."

d. "Combination therapy attack the parasite at different stages of its life cycle." (Rationale: Combination therapy is used to attack the parasite at various stages of its life cycle. Quinine [Qualaquin], the first drug found to be effective against malaria, is no longer available. More than one drug is usually needed to prevent resistance to the single drug. Many strains of the parasite are developing resistance, especially to chloroquine.)

The nurse is administering anidulafungin to a patient via an IV infusion for the treatment of candidemia. The nurse would anticipate that this therapy would be given for how many days? a. 21 days b. 28 days c. 7 days d. 14 days

d. 14 days (Rationale: Anidulafungin as treatment for candidemia is administered for 14 days. Treatment for only 7 days could result in development of a medication-resistant infection. Treatment is usually shorter than 21 days.)

A nurse is monitoring a patient's serum flucytosine level. Which results would cause the nurse the least amount of concern? a. 125 mcg/mL b. 150 mcg/mL c. 200 mcg/mL d. 75 mcg/mL

d. 75 mcg/mL (Rationale: Flucytosine toxicity is associated with serum levels greater than 100 mcg/mL. Thus a level less than 100 mcg/mL would be of the least concern. A level above 100 mcg/mL would be of concern and could indicate toxic levels.)

The nurse is caring for a patient diagnosed with a life-threatening systemic fungal infection that is not responding to treatment with an azole. What medication will the nurse anticipate the provider may order next? a. Nystatin b. Terbinafine c. Butenafine d. Amphotericin B

d. Amphotericin B (Rationale: Amphotericin B is a potent drug with many serious adverse effects so it is often used only when other medications are not effective. Nystatin and butenafine are topical agents and would not be used for a systemic infection. Terbinafine is an azole and would not be used if azoles do not work.)

After teaching a group of students about antineoplastic antibiotics, the instructor determines that the teaching was successful when the students identify which drug as an example? a. Methotrexate b. Imatinib c. Vincristine d. Doxorubicin

d. Doxorubicin (Rationale: Doxorubicin is the prototype antineoplastic antibiotic. Vincristine is the prototype mitotic inhibitor. Imatinib is the prototype protein tyrosine kinase inhibitor. Methotrexate is the prototype antimetabolite.)

An instructor is describing the action of primaquine. What would the instructor include? a. The drug blocks the use of folic acid. b. It changes the metabolic pathways for reproduction. c. The drug increases the acidity of plasmodial food vacuoles. d. It disrupts the mitochondria, killing the gametocytes.

d. It disrupts the mitochondria, killing the gametocytes. (Rationale: Primaquine disrupts the mitochondria and causes death to the gametocyte and exoerythrocytic forms. Mefloquine increases the acidity of plasmodial food vacuoles, causing cell rupture and death. Pyrimethamine blocks the use of folic acid in protein synthesis by the plasmodium. Chloroquine changes the metabolic pathways necessary for reproduction of the plasmodium.)

During which phase of the cell cycle would a nurse expect a mitotic inhibitor to act? a. G1 b. G0 c. S d. M

d. M (Rationale: Mitotic inhibitors act only during the M phase of the cell cycle.)

A patient is treating a skin mycosis with a topical antifungal agent in troche form. The nurse would alert the patient to what possible adverse effect? a. Burning on urination b. Rash c. Urinary frequency d. Nausea

d. Nausea (Rationale: Nausea may occur with troche use due to absorption of some of the drug by the GI tract. Burning on urination or increased urinary frequency could occur if the agent was administered as a vaginal suppository, cream, or gel. Rash would most likely be associated with application as a cream, lotion, or spray.)

An antineoplastic extravasates and the antidote is administered. The nurse would apply warmth to the site of extravasation if what drug was involved? a. Dactinomycin b. Doxorubicin c. Mechlorethamine d. Teniposide

d. Teniposide (Rationale: Heat/warm compresses are applied if teniposide extravasates. Ice compresses are applied if mechlorethamine extravasates. Cold compresses are applied if doxorubicin extravasates. Cold compresses are applied if dactinomycin extravasates.)

intradermal

dermal layer, under epidermis 1 mL syringe 10-15 degree angle

D

dextrose

What physical characteristic observed in an adult suggests that they were prescribed tetracycline as a child?

discoloration of the teeth

agents for influenza A adverse effects

dizzy insomnia nausea orthostatic hypotension urinary retention

CCR5 co. antagonist adverse effects

dizzy, changes in consciousness

In the event of a bioterrorism attack with anthrax, the health care team should anticipate the use of which tetracycline?

doxycycline

DF

drop factor

gtt/mL

drops per milliliter

Dial-a-Flow

extension IV tubing that attaches to the primary IV tubing (calibrated in mL/hr)

how to measure using medicine cup

eye level measure at meniscus level (curved part)

Order: Zithromax oral susp 400 mg Supply: 5 mL Have: 200 mg *before solving each problem, check to make sure that the order and supply are in the same measurement. If they are not, you must convert one or the other to it's equivalent. Convert whichever is easiest for you to solve*

give 10 mL 400 mg / 200 mg x 5 mL = 10 mL

Example: Order: 1 g IVPB Supply: Package insert IVPB dilution. Reconstitute with 50 to 100 mL of sodium Cl injection. Use a 50 mL bag of D5W. No time was given so use 30 min for 50 mL.

give 17 gtt per min 50 mL x 10 ------------ = 17 gtt/min 30 mintues

A nursing student is reading a journal article about highly active antiretroviral therapy (HAART). The nurse would expect to find which condition as being treated with this therapy?

human immunodeficiency virus (HIV)

CCR5 co. antagonist contraindications

hypersensitivity nursing mothers liver disease

integrase inhibitor contraindications

hypersensitivity nursing mothers

NRTI adverse effects

hypersensitivity, pancreatitis, hepatomegaly, neurological problems, bone marrow suppression

A nurse is caring for a patient, who has been prescribed the antiviral drug amantadine. Which of the following adverse reactions specific to amantadine should the nurse monitor for?

hypotension and insomnia

Converting Order and Supply to the SAME WEIGHT MEASURE

if the order and supply are in a differing weight measure, then you must convert one or the other amount to it's equivalent. For example if the Order states 1 g and the drug is supplied in mg, then a conversion is needed from grams to milligrams or milligrams to grams.

The client has been prescribed a fluoroquinolone. The nurse knows that nursing interventions for clients taking fluoroquinolones include which?

increase fluid intake.

Macrodrip

infusion sets without a small needle in the chamber 10gtt/mL 15gtt/mL 20gtt/mL

A neonatal intensive care unit nurse is caring for an infant diagnosed with respiratory syncytial virus (RSV). What route of delivery will the nurse use when ribavirin is administered to this infant?

inhaled

When describing the action of sulfonamides to a client, the nurse integrates understanding of the drug's action with effects exerted due to which mechanism?

inhibition of the activity of folic acid in bacterial cell metabolism

Bone marrow suppression

inhibition of the blood-forming components of the bone marrow; a common adverse effect of many antineoplastic drugs, which are more effective against rapidly multiplying cells, such as those in bone marrow

subcutaneous

injected into fatty tissue, 45 degree angle (depending on pt size)

fusion inhibitor adverse effects

insomnia, depression, peripheral neuropathy, nausea, diarrhea, pneumonia, injection site reactions

A group of nursing students are researching information about antiretroviral agents. They find information about a category of drugs that prevent enzymes from integrating HIV genetic material into the host cell's DNA. The students are reviewing information related to which agents?

integrase inhibitors

parenternal route

intradermal, subcutaneous, intramuscular, intravenous

When a person is diagnosed with tuberculosis, it is most likely that the nurse prepares the family members to be treated prophylactically with which drug?

isoniazid

Order: 500 mg in 500 mL D5W IV at 50 mL/hour

it will run for 10 hours 500 mL ---------- = 10 Hours 50 mL in this example the amount of drug is not needed for the calculation of IV Rate

CCR5 coreceptor antagonist

maraviroc

oral medication delivery

medicine cups, droppers, oral syringes, medicine spoon (infant &toddlers)

right time

meds can be given 30 minuets before or after the ordered time

The nurse is justified in suspecting that a client who recently completed a course of ceftaroline may have been treated for what health problem?

methicillin-resistant Staphylococcus aureus (MRSA) infection

What is amebiasis?

mild, abdominal discomfort and bloody diarrhea with alternating periods of consitpation and remission

Pre-mixed insulin

novolin 70/30 contains 70% NPH and 30 % regular insulin

determining hours an IV will run

number of milliliters ordered/number of milliliters per hour= number of hours to run

What are common adverse effects of antiprotozoals

ocular damage CNS effects dermatological effects GI effects

Some are selective -

only effective for a few number of organisms

Order: Zyprexa 7.5 mg by mouth everyday Supply 1 tablet Have: 5 mg per tablet

only give 1 and a half of a tablet 7.5 mg / 5 mg x 1 tablet = 1 1/2

Order: Lanoxin 0.125 mg Supply: 1 Tablet Have: 0.25mg

only give 1/2 a tablet 0.125 mg / 0. 250 mg x 1 tablet = 1/2

A client comes to the clinic for a follow-up visit. The nurse notes a sunburn-like appearance to the client's skin. The client's history reveals the use of ciprofloxacin. The nurse interprets which adverse effect regarding this finding?

photosensitivity

Pregnant and nursing women should only take antiprotozoals when benefit outweighs risk. A traveling pregnant women should be warned about serious risks to fetus associated with both what?

preventative therapy and effects that an infection could have

A nursing student studying pharmacology is focusing on drugs used to treat tuberculosis (TB). This student correctly identifies the classifications of antitubercular drugs as which of the following?

primary and secondary

amounts less than 1 mL

round to hundredths place, use 1 mL

for amounts greater than 1 mL

round to tenths place, use 3 mL syringe

A client with a diagnosis of rhinosinusitis has been prescribed ciprofloxacin 250 mg SC b.i.d. When contacting the prescriber, the nurse should question the:

route

Example: Order: Infuse 120 mL of Iv Fluid over 60 mins. Infusion pump Included

run 120mL/hr. 120 mL --------- = 120 mL 1 hour (60 min= 1 hour)

Leishmaniasis is caused by what

sand flies

When should Amphotericin B be used? What form is it given in?

serious fatal infections IV form

insulins that are often mixed

short-acting and intermediate (regular and NPH)

What is a protozoa?

single cell organism that passes through several stages in their life including at least ONE phase as a HUMAN PARASITE

A nurse is speaking to a 62-year-old female client who has been started on sulfisoxazole, a sulfonamide antibiotic. The nurse should teach this client to contact the health care provider if the client experiences what adverse effect associated with the drug?

skin rash or itching

IV fluids are packaged in

sterile plastic bags glass bottles

how can intermediate insulin be givin

subcutaneous only

how can long actin insulin be given?

subcutaneous only

how can rapid acting insulin be given?

subcutaneously and intravenously

how can short acting insulin be given

subcutaneously and intravenously

A client has been treated several times for a recurring ear infection with ofloxacin. The nurse instructs the client to look for side effects from the prolonged use of the antibiotic and informs the client that this is known as:

superinfection

NRTI drug drug i

tenofovir lamivudine antibiotics antifungals

What antibiotics may be administered in treatment with Quinine when a strain is found to be resistant to Chloroquine?

tetracycle doxycycline clindamycin

The nurse is caring for a 3-year-old with HIV. The nurse knows that, when administering antiviral drug therapy in young children with HIV, dosage calculations are typically based on:

the client's weight

What group of antifungals inhibit glucan synthesis and are used to treat more serious infections

the echinocandins

A 38-year-old man is being treated for HIV-1 with nevirapine (Viramune). Signs of severe liver dysfunction have been noted by the health care provider and a change in the drug therapy is necessary. The nurse explains to the client that:

the therapy will be discontinued

Example: Order: Infuse 120 mL of IV Fluid over 60 minutes with a tubing factor of 10 drops per mL. solve this calculation.

there are 20 gtt/min 120 mL x 10 gtts ------------------- = 20 gtt/min 60 mintues

Topical antifungals treat ________ infections

tinea

Solving IV calculations Using an Infusion Pump

total number of mL ordered -------------------------------- = mL/hr. number of hours to run

solving IV calculations using an infusion pump

total number of milliliters ordered/number of hours to run= mL/hour

What is prophylaxis?

use of a drug to prevent infection of a person at risk

parenternal medications

vials, ampules, mix-o-vial, pre-filled syringes

What would the nurse recognize as the primary goal(s)of highly active antiretroviral therapy (HAART)? Select all that apply

• Suppress virema. • Delay drug-resistant mutations. • Preserve CD4 counts.

When focusing on the lifestyle of the patient with HIV, what are important teaching considerations? (Select all that apply.)

• Take drug therapy as prescribed. • Transmission can occur while on medication. • Advise the patient about the adverse effect of granulocytopenia.

Your patient is HIV positive and is involved in ART to decrease symptoms and increase lifespan and quality. The regimen currently involves a combination of antiretrovirals. The rationale to support this treatment regimen is: (Select all that apply.)

• To increase drug efficacy. • To decrease viral mutations.

Place the stages of the replication cycle of a virus in the correct order.

• Virus adheres to host cell surface • Virus enters host cell by pinocytosis • Virus sheds its coat • Viral nucleic acids replicate • Synthesis of viral protein capsid • Assembly of new virions and Host cell dies releasing virus to move to other cells

Administration of antiviral therapy for influenza requires monitoring of clients for which adverse effects? Select all that apply.

• anorexia • nausea • dizziness

A client has sought care from her primary care provider after "feeling under the weather" for several days. What signs and symptoms would support the presence of a viral infection? Select all that apply.

• malaise • fever • headache

A group of nursing students are reviewing information about the different antiretroviral drugs. The students demonstrate understanding when they identify which as an example of an entry inhibitor? Select all that apply.

• maraviroc • enfuvirtide

When assessing a client prescribed lamivudine therapy for adverse effects, which assessment should the nurse prioritize? Select all that apply.

• nerve pain • muscular pain • pancreatic function • gastrointestinal distress

A child is prescribed ribavirin for a diagnosis of respiratory syncytial virus (RSV). The nurse should the nurse assess the client often for which possible adverse reaction? Select all that apply

• rhinitis • headache • bronchospasm • conjunctivitis


Conjuntos de estudio relacionados

QUIZ 2 (only a few) Patho practice questions-renal

View Set

Chapter 7 - Advanced Regression Analysis

View Set

Fear Avoidance Beliefs Questionnaire (FABQ)

View Set

STIs chapter 71 infectious diseases

View Set

Neurologic and Sensory Systems Level 1 Questions

View Set